Этого треда уже нет.
Это копия, сохраненная 23 декабря 2017 года.

Скачать тред: только с превью, с превью и прикрепленными файлами.
Второй вариант может долго скачиваться. Файлы будут только в живых или недавно утонувших тредах. Подробнее

Если вам полезен архив М.Двача, пожертвуйте на оплату сервера.
Vopreki-svoey-prirode-chernyie-dyiryi-izluchayut-chastitsyi145 Кб, 1300x731
Глупых вопросов тред Сжигатель !HOHLOVz6ow #413326 В конец треда | Веб
Здесь ты можешь написать глупый вопрос, но не глупый ответ.
#2 #413369
1) Для чего нужна смерть?
2) Какова вероятность, что белые дыры действительно существуют?
3) Как скоро (хотя бы примерно) произойдёт создание теории всего?
#3 #413370
Стоит ли обмазываться квантовой механикой или лучше подождать Теории всего?
#4 #413372
>>413369

> 1) Для чего нужна смерть?


СНН
#5 #413374
А вот грят, метан/гелий вытесняет кислород из воздуха - именно кислород или всё остальное тоже или кислород+ещ некоторые газы?
#6 #413393
Вопрос по крипте:
Вот есть такие вещи, как MAC, HMAC или ЭЦП. А как принято называть последовательность действий, когда ключ формируется на основе хэша от сообщения, этим ключом шифруется сообщение, а сам хэш отправляется вместе с сообщением, чтобы получатель смог сгенерировать ключ для дешифрования на основе этого хэша и своего ключа? (https://core.telegram.org/mtproto/description)
#7 #413395
>>413369
1) Для чего нужна смерть?
Чтобы был выбор. А вот старость не нужна.
1113881061381000134 Кб, 1000x663
#8 #413397
>>413326 (OP)
Как заебала эта фальсифицируемость. Неужели на моём веку не изобретут что то новое? Какое нибудь трансендентное понимание сути вещей в себе напрямую. Если бы я уснул и проснулся через миллион лет а люди всё еще дальше модельнозависимого реализма не продвинулись то я бы выпилился от разочерования.
S70525-2313593949 Кб, 839x741
#9 #413399
Все знают историю про два корабля, где из одного летит световой луч на другой корабль.
Рассмотрим эту ситуацию относительно стороннего наблюдателя. Везде пишется, что этот луч будет двигаться, как желтый на данной пикче и именно поэтому время замедляется, а свет проходит большее расстояние за одинаковую скорость.
Но почему луч будет двигаться именно так? Почему он не будет двигаться, как красный и тогда наблюдатель не увидит, как луч попадает на второй корабль?
#10 #413400
>>413397

>разочерования


Приступай.
#11 #413401
>>413399
Чё за история? В какую сторону гуглить?
#12 #413403
>>413399
Я тогда нарушится причина и следствие, на корабле стреляют лучем в другой корабль(который напротив, и не движется) А я вижу будто он промахивается.
#13 #413406
Что представляет собой цифровой и аналоговый сигнал? Ведь по сути и там и там ток . нихрена непонятно.
#14 #413407
>>413401
Специальная теория относительности
#15 #413409
>>413406
Прерывность.
Hanyuupersonnality18 Кб, 360x344
!My/HanyurE #16 #413411
Как можно описать синий цвет, кроме как сказать, что это синий цвет?
Palanca-ejemplo33 Кб, 888x398
#17 #413415
Почему работает рычаг? Ведь в любом случае тяжелое должно перевешивать, хули на уровне гравитации происходит?
#18 #413416
>>413411
Длиной волны.
LeverPrincleple[1]4 Кб, 500x300
#20 #413419
>>413415
Схема рычага.
В равновесии: F1×D1 = F2×D2;
F1 = m1×g; F2 = m2×g;
m1×g×D1 = m2×g×D2 | /g
m1×D1 = m2×D2;
100×1 = 20×5 = 100 - рычаг в равновесии.
#21 #413420
Хуй растянет в черной дыре
#22 #413421
>>413420
это вопрос или ответ?
#23 #413422
#24 #413426
>>413369
Не нужна.

>какого вероятность


Крайне мала, но 100% уверен что 100 лет они будут обыденностью. Вселенная тролит хомячье.

>как скоро


Давно уже есть теории, ктож виноват что ты жопочтец или не чтец вообще.
#25 #413442
Аноны, а чем усталость от отсутствия сна (накопление продуктов белкового распада(?) в межклеточном пространстве мозга) отличается от усталости, связанной с истощением нейромедиатора(заработался человек)? Хотелось бы линков на статьи с этими механизмами ну или еще чего либо в этом духе. Хочу понять когда и как надо правильно отдыхать.
#26 #413443
>>413326 (OP)
Почему водородные связи особенные? Почему нету в таком случае хлорных, кислородных, натриевых и т.д. связей?
#27 #413446
>>413326 (OP)
Я пишу научную фантастику и мне нужна консультация по поводу некоторых вещей относительно возможных технологий будущего.

Конкретно сейчас интересует наиболее реалистичное описание механизма искусственной гравитации. Допустим, я хочу ввести некое вещество, у которого инертная масса будет сильно меньше массы гравитационной. Возможно ли такое теоретически? Как такой объект будет себя вести?
#28 #413447
>>413446

>инертная масса будет сильно меньше массы гравитационной


Это одно и то же на уровне ОТО, так что такого быть не может.
#29 #413448
>>413447
А как тебе такое: генератор некоторого альтернативного силового поля, который притягивает всё, что в этом поле находится. Ну как магнитное поле, только действующее не на одни заряды. При этом такое поле может иметь свою форму и не иметь связи с массой и обычной гравитацией.
#30 #413454
>>413448
Да ради бога. Я конкретно по теории поля не специалист, и мои познания по ней заканчиваются на прослушанном университетском курсе и прочитанном втором томе ландавшица. Поэтому здесь мне сложно сказать, где у тебя есть свобода.

Если бы ты спросил, как лично я отношусь к такой задумке, то я сторонник твердого сай-фая, поэтому мне бы такое не очень зашло. Такие моменты -- это магия и это слишком просто для писателя. В то же время, когда известные технологии и физические законы как-то с изъебом обыгрываются, выглядит это более оригинально.
много рыбы19,3 Мб, webm,
400x304
#31 #413481
Почему морские рыбы не могут жить в пресной воде и наоборот?
#32 #413482
>>413369
Чтобы не было перенаселения.
#33 #413486
>>413481
Пресноводные животные, погруженные в морскую воду, умирают вследствие экзосмотического действия соленой воды на кровь и другие жидкости их тела. У животных, кожа которых покрыта предохраняющей слизью, соленая вода действует главным образом через жабры: тонкая кожица их тускнеет, кровообращение останавливается. У животных, покрытых гладкой, тонкой кожей, без слизи (у лягушек и их головастиков) экзосмоз совершается через всю поверхность тела; животное "высыхает": соленая вода извлекает воду из крови и тканей, и лягушка умирает, теряя от 1/4 до 1/2 своего веса. Можно, таким образом, убить лягушку, погрузив только одну ее лапку в морскую воду.

Для морских животных, погруженных в пресную воду, причиной смерти является потеря хлористого натрия; пресная вода извлекает его из их тканей. К воде добавляли раствор других солей и веществ (сахара, глицерина) с целью увеличить ее плотность до степени плотности морской воды, но это нисколько не защищало животных от быстрой гибели. Пресная вода действовала обратно, эндосмотически, в избытке проникая в ткани и кровь морских животных: у рыбы раздувались жабры и в них останавливалось кровообращение, у низших животных прозрачный эпителий становился тусклым, мышцы теряли свою сократимость.

В процессах выделения и водно-солевого обмена кроме почек принимают участие кожа, жаберный эпителий, пищеварительная система. Осмотическое давление водных животных создается давлением их полостных жидкостей, давлением крови и соков тела. Определяющая роль в этом процессе принадлежит водно-солевому обмену. Каждая клетка тела имеет оболочку: она полупроницаема, т. е. по-разному проницаема для воды и солей (пропускает воду и соль избирательно) . Водно-солевой обмен клеток определяется в первую очередь осмотическим давлением крови и клеток.
У морских костистых рыб общее количество солей в крови значительно ниже, чем в морской воде, давление внутренней среды меньше давления внешней, т. е. их кровь гипотонична по отношению к морской воде.
У пресноводных рыб количество солей в крови выше, чем в пресной воде. Давление внутренней среды больше давления внешней, их кровь гипертонична.
Во избежание чрезмерного обводнения, для сохранения водно-солевого состава и уровня осмотического давления возникает необходимость вывода из организма лишней воды и одновременного удержания солей. В связи с этим у пресноводных рыб мощное развитие получают почки. Утрата солей с мочой, экскрементами и через кожу восполняется у пресноводных рыб за счет получения их с пищей благодаря специализированной деятельности жабр (жабры поглощают из пресной воды ионы Na и Сl) и поглощением солей в почечных канальцах.
Морские костистые рыбы (с гипотонической кровью) , находящиеся в гипертонической среде, постоянно теряют воду – через кожу, жабры, с мочой, экскрементами. Предотвращение обезвоживания организма и сохранение осмотического давления на нужном уровне (т. е. ниже, чем в морской воде) достигаются тем, что они пьют морскую воду, которая всасывается через стенки желудка и кишечника, а избыток солей выделяется кишечником и жабрами.
#33 #413486
>>413481
Пресноводные животные, погруженные в морскую воду, умирают вследствие экзосмотического действия соленой воды на кровь и другие жидкости их тела. У животных, кожа которых покрыта предохраняющей слизью, соленая вода действует главным образом через жабры: тонкая кожица их тускнеет, кровообращение останавливается. У животных, покрытых гладкой, тонкой кожей, без слизи (у лягушек и их головастиков) экзосмоз совершается через всю поверхность тела; животное "высыхает": соленая вода извлекает воду из крови и тканей, и лягушка умирает, теряя от 1/4 до 1/2 своего веса. Можно, таким образом, убить лягушку, погрузив только одну ее лапку в морскую воду.

Для морских животных, погруженных в пресную воду, причиной смерти является потеря хлористого натрия; пресная вода извлекает его из их тканей. К воде добавляли раствор других солей и веществ (сахара, глицерина) с целью увеличить ее плотность до степени плотности морской воды, но это нисколько не защищало животных от быстрой гибели. Пресная вода действовала обратно, эндосмотически, в избытке проникая в ткани и кровь морских животных: у рыбы раздувались жабры и в них останавливалось кровообращение, у низших животных прозрачный эпителий становился тусклым, мышцы теряли свою сократимость.

В процессах выделения и водно-солевого обмена кроме почек принимают участие кожа, жаберный эпителий, пищеварительная система. Осмотическое давление водных животных создается давлением их полостных жидкостей, давлением крови и соков тела. Определяющая роль в этом процессе принадлежит водно-солевому обмену. Каждая клетка тела имеет оболочку: она полупроницаема, т. е. по-разному проницаема для воды и солей (пропускает воду и соль избирательно) . Водно-солевой обмен клеток определяется в первую очередь осмотическим давлением крови и клеток.
У морских костистых рыб общее количество солей в крови значительно ниже, чем в морской воде, давление внутренней среды меньше давления внешней, т. е. их кровь гипотонична по отношению к морской воде.
У пресноводных рыб количество солей в крови выше, чем в пресной воде. Давление внутренней среды больше давления внешней, их кровь гипертонична.
Во избежание чрезмерного обводнения, для сохранения водно-солевого состава и уровня осмотического давления возникает необходимость вывода из организма лишней воды и одновременного удержания солей. В связи с этим у пресноводных рыб мощное развитие получают почки. Утрата солей с мочой, экскрементами и через кожу восполняется у пресноводных рыб за счет получения их с пищей благодаря специализированной деятельности жабр (жабры поглощают из пресной воды ионы Na и Сl) и поглощением солей в почечных канальцах.
Морские костистые рыбы (с гипотонической кровью) , находящиеся в гипертонической среде, постоянно теряют воду – через кожу, жабры, с мочой, экскрементами. Предотвращение обезвоживания организма и сохранение осмотического давления на нужном уровне (т. е. ниже, чем в морской воде) достигаются тем, что они пьют морскую воду, которая всасывается через стенки желудка и кишечника, а избыток солей выделяется кишечником и жабрами.
#34 #413487
>>413369
Даю тебе ответы, не знаю, нахуй они тебе нужны.

>1) Для чего нужна смерть?


У жизни нет смысла, ее нет и у смерти.

>2) Какова вероятность, что белые дыры действительно существуют?


Никто не знает, существуют ли черные дыры.

>3) Как скоро (хотя бы примерно) произойдёт создание теории всего?


Это невозможно. Теоретические поиски в отдельных отраслях, как то в программировании или в математике, привел изследователей к факту невозможности создания всезнающих программ или всеобъемлющих математических теорий. Это же позволяет говорить о божественой воле, поскольку, следуя расширеной теории невозможности всеобъемлющести, нет ничего материального, что могло бы выйти за свои рамки, однако, оно возникает.
#35 #413495
Есть стальная деталь. Если ее резко, за 1-2 секунды охладить градусов на 150-200(не спрашивай как), придет ли ей пиздец?
135405534561129 Кб, 500x328
#36 #413496
Поясните как работает наклонная плоскость.
1) Профит возникает из-за того, что мы не поднимает тело целиком, а кладём его на плоскость и просто тащим, таким образом оно меньше весит = меньше силы нужно?? Но увеличивается расстояние, и как посчитать в итоге мы сэкономили энергии или нет?
2) Ещё получается эффективность наклонной плоскости зависит от угла, длины, гладкости и что по ней тащат. Я представил например что мы берём плиту очень большой площади и пытаемся затащить, но по идее в этом случае трение будет настолько велико, что проще поднять. Но затем я иду на вики и вижу

>Так как никакое тело не является абсолютно ровным, сила трения не зависит от площади соприкосновения


И как это понимать? А сила трения вообще только в формулах существует, её же нечем измерить. Все эти задачки из учебников в итоге далеки от реальности. В реальности без погрешности нельзя описать даже все переменные когда мы трём одну доску о другую?

3) Почему лестницу считают за наклонную плоскость? Она же не сплошная и каждая ступенька горизонтальная. Да и вообще можно ли считать человека за тело которое тащат, если из-за строения скелета ему удобней одной и неудобно другое.
#37 #413498
>>413326 (OP)
Потенциальная яма = отрицательная температура ниже абсолютного нуля? При температуре больше нуля кельвин тело движется, при нуле стоит но с маленького милипизерного пинка начинает двигаться, при отрицательной логично тело должно и дальше оставаться на месте и чтобы его заставить двигаться надо потратить тем больше энергии чем выше(или ниже? кароче дальше от нуля) отрицательная температура, а чем глубже потенциальная яма, тем больше надо энергии затратить что бы тело начало двигаться, вроде все верно, да и потенциальные энергии вроде как правило часто со знаком минус. В чем ошибка мои рассуждений?
image44 Кб, 472x381
#38 #413500
>>413498
У любой потенциальной ямы есть дно, в виде нуля энергии. В виде пустого пространства например, температура вводилась в виде движения молекул 150 лет назад. Сейчас есть квантовая температура, которая обобщается на термодинамическую, с помощью статистики. И да, отрицательную можно хитровыебано получить, но стоит понимать что температура, это просто статистическая величина.
#39 #413505
>>413496

>Поясните как работает наклонная плоскость.


Как рычаг. Пикрелейтед.
Энергия та же самая нужна, ведь потенциальная энергия равна Ep = mgh;

>Так как никакое тело не является абсолютно ровным, сила трения не зависит от площади соприкосновения


Ты выдернул из контекста, основное дальше:
...и истинная площадь соприкосновения гораздо меньше наблюдаемой;
кроме того, увеличивая площадь, мы уменьшаем удельное давление тел друг на друга.

>В реальности без погрешности нельзя описать даже все переменные когда мы трём одну доску о другую?


Да причём тут переменные, просто разница берётся, усредняется и всё.
Можешь смотреть на природу силы трения как на принцип работы липучки. Пик2, пик3.
Только там молекулы и силы Ван Дер Ваальса зависят от давления и площади соприкосновения.

>Почему лестницу считают за наклонную плоскость? Она же не сплошная и каждая ступенька горизонтальная.


Наверное из-за трения об ступеньки?!.
#40 #413506
>>413326 (OP)
Что внутри черной дыры? Ведь это сфера вполне определенного диаметра, должно быть внутреннее пространство.
#41 #413507
А скажите, вот в вк есть фотки голых девушек, всякие модели из питера и москвы. Скажите, они фотаются голыми и еще сами за это платят, за то чтобы их фотали, они еще и за это платят фотографам, которые мало того, что дрочат на них так еще и бабло получают за фотки? Просто я так понимаю услуги фотографа платные. И зачем тогда девушка платит деньги чтобы ее сфотали да еще и голой? Это же бред какой-то. Соре, если не в том разделе, но просто я пытаюсь понять как устроен мир на макроуровне, а не зашкварки и электроны.
#42 #413508
>>413506
Да ничего там нет. Более того черных дыр нет. Как бы еще не доказано, но да объекты похожие существуют и которые математически описываются как черные дыры. Имхо, но это просто какой-то объем огромной массы и все. Ну и вокруг этого центра масс вертятся звезды, а в центре галактики суперогромная масса. Вон темная материя тоже свет не излучает. А ведь она очень массивна. Ничо никуда там не падает и ничего внутри нет. ну или там выход через белую дыру в другую вселенную, но тогда ты в нашу уже не вернешься
#43 #413511
>>413508
Так если внутри ничего нет и ничего внутрь не попадает, так почему же ЧД не обрастает коркой материи, притянутой гравитацией? Куда звездное вещество девается?
#44 #413517
>>413505

>Как рычаг. Пикрелейтед.


Либо ты что-то путаешь, либо я не понимаю. Что общего между наклонной плоскостью и рычагом? Это два разных механизма. Я про плоскость спрашиваю. Просто везде в роликах принцип работы объясняют формулами, что мне не нужно. По наклонной плоскости легче тащить предмет потому что часть его веса распределяется по плоскости, и от этого профит. Это я прально понимаю? А углом наклона мы изменяем то сколько будет весить тело и соответственно силу которую нужно к нему приложить. Плюс сюда замешана сила трения, которая может и не давать по итогу никакого выигрыша в силе.
Но механическая работа по поднятию груза без плоскости и с ней будет одинакова?
#45 #413518
>>413517
Ну, в рычаге, по определению подобия треугольников - соотношение между длинами плеч определяет соотношение между силами:
>>413415>>413419
А здесь, на этой наклонной плоскости - тоже соотношение, но между разными катетами и гипотенузой,
потому что синус и косинус там фигурируют в формулах.

>Это два разных механизма. Я про плоскость спрашиваю.


Отношение между катетами определяет тангенс угла, а значит и длину гипотенузы,
и путь, который проходит груз и силу, которую нужно приложить для того чтобы сообщить ему в момент подъёма энергию mgh.
И как-то вот так вот, у меня, эта наклонная плоскость, всё-же таки-смахивает на рычаг, лол.

>По наклонной плоскости легче тащить предмет потому что часть его веса распределяется по плоскости, и от этого профит.


>Это я прально понимаю?


Но ведь от площади зависит и суммарная сила трения, потому что сцепление с поверхностью там,
когда площадь растёт - увеличивается. В то же время при большой площади уменьшается давление, а значит и сила трения.
Подумай над тем - стоило бы плоский лист гипсокартона на колёсиках поднимать по каким-нибудь шершавым волосам?
С одной стороны, при маленьком всё это легче скатывалось бы вниз, а значит силу надо было бы приложить больше.
С другой стороны, трение препятствует поднять груз.
Можно было бы однонаправленное трение вниз сделать, как например у колоска из-за его упорядоченных ворсинок.

>Но механическая работа по поднятию груза без плоскости и с ней будет одинакова?


Да. Вверху тело обладает потенциальной энергией. Её величина равна Ep = mgh, где m - масса тела, g - ускорение свободного падения, h - высота.
Этой энергией будет обладать любое тело с такой массой, поднятое на такую высоту.
И чтобы поднять тело, нужно эту энергию приложить туда.
Наклонная плоскость, как и рычаг - позволяет уменьшить силу за счёт увеличения пути, по сравнению с высотой, но не энергию конечно же.
#46 #413520
>>413517

>Что общего между наклонной плоскостью и рычагом? Это два разных механизма.



Вот тебе короче, сварганенный по быстрячку рычаг на плоскости.
Чтобы поднять груз на одно деление, в зависимости от угла наклонной плоскости, разные грузы проходят разное расстояние также как и в обычном рычаге.
richag177 Кб, 1274x673
#47 #413526
>>413518

>соотношение между длинами плеч определяет соотношение между силами


>А здесь, на этой наклонной плоскости - тоже соотношение, но между разными катетами и гипотенузой


Вот в этом проблема. Я прошу объяснений принципа работы механизмов, а не геометрических закономерностей которые в них нашли. Ведь не длина делает работу, а сила (вес).
И насколько я понял в :
Наклонной плоскости — мы получаем выигрыш в силе т.к часть веса опирается на плоскость. То есть мы не поднимаем его вертикально, справляясь с полным весом (m*g), а лишь частично, то есть заменяем часть веса на силу трения (если оно есть) осталось разобраться что такое вес и масса не свихнувшись

Рычаг — мы получаем выигрыш в силе из-за разностей центра тяжести. Мы смещаем центр тяжести и он просто работает на нас. Корректировкой длины плеча мы выбираем эффективность рычага.

Вот я это узнать пытаюсь. А не геометрические модели, какие-то соотношения. Это всё вторичные вещи. Почему саму суть нигде не пишут? Алсо гляньте вот этот пример.
https://www.youtube.com/watch?v=2RPuG362_mI
5:08
Мне кажется они в видео сделали ошибку. И сами же со своей геометрией облажались. Если мысленно обрезать балку то видно сколько делений уравновешивает груз. Они неверно определили плечо и вес соответственно. Мужик в коментах математикой это заметил, но вроде тоже ошибся.
#48 #413530
>>413526

>Ведь не длина делает работу, а сила (вес).


Внезапно работа зависит от длины >:-)

>мы получаем выигрыш в силе т.к часть веса опирается на плоскость.


Тоже правильно. Хотя отсюда не очевидна ситуация с работой и энергией.

>Рычаг — мы получаем выигрыш в силе из-за разностей центра тяжести.


А вот это уже бред...
Проводя аналогию с плоскостью, рычаг нуждается в точке опоры, на которую опирается, что и есть ещё одна сила, помогающая в данном случае. Если с помощью рычага поднимается груз, то любопытно, что силу к другому концу рычага надо прикладывать по направлению вниз. (сравнить с плоскостью, простым подъёмом и удивиться).

//Мимо один один...
#49 #413531
>>413506
Там внутри что-то особенного. Хоть и не всё. То есть вроде бы частью ничего особенного и почти как обычно, и ещё вроде бы есть кое что ещё - Сингулярность. И ещё там внури происходит большое схлопывание маленькой внутренней вселенной... :-(

>>413508

>Более того черных дыр нет. Как бы еще не доказано, но да объекты похожие существуют и которые математически описываются как черные дыры.


Те которые похожи - они просто похожи... Но даже если чёрных дыр и не существует, вопрос имеет смысл...
#50 #413532
>>413530

>Внезапно работа зависит от длины


Да, только в физике под длиной надо подразумевать количество вещества которое имеет свою массу и кучу всевозможных характеристик.
В то время как в геометрии длина - это просто абстракция. То есть надо понимать что лежит под этими сантиметрами, а не объяснять всё ими.

>А вот это уже бред...


Я ещё не смотрел все виды рычагов, типа блоков. Если брать обычный рычаг: подвесной или на опоре - то вся хитрость в том, что мы смещаем опору (ось) как можно ближе к телу которое хотим поднять. Таким образом центр тяжести находится почти что на самой оси, это как монету на ребро поставить. Дальше мы давим на плечо и там работает сопромат, сопротивление материала ещё больше усиливает эффект рычага. Понятно что из эластичного материала рычаг будет не эффективный.
В итоге формула рычага: центр тяжести над осью + напряжение которое возникает в самом рычаге. А если ты объясняешь чисто геометрически типа длина плеча обратно пропорциональна силе - то надо понимать, что это работает не потому что сантиметры увеличились, а потому что стало больше материала и сила сопротивления возросла. Да и вообще если рычаг не однородный то и формулировка эта работать не будет.
#51 #413533
Почему параллейные измерения так похожи на наше?
#52 #413539
>>413533

>параллейные


Параллельные.

>на наше


на наши.

Какие такие измерения, 11-мерной М-теории начитался что-ли?
#53 #413540
>>413539
Трехмерные параллельные нашему измерению измерения.
arch[1]5 Кб, 258x152
#54 #413544
>>413530

>Вот в этом проблема.


Я тебе уже пояснил чем наклонная плоскость смахивает на рычах, и даже нарисовал таки-рычаг из этой наклонной плоскости.
Ты что ещё не понял что-ли? Что ты не понял там?

>Ведь не длина делает работу, а сила (вес).


Работу по подъему груза делает разница сил, которая определяется силой f,
приложенной для подъема груза, и силой, которая тянет груз вниз - mg×sin(α);
Исходя из первой картинки >>413505
И величина этой силы прямо пропорциональна значению угла в градусах,
т. е. чем этот угол больше, и стремится к прямому, тем больше сила.
Чем меньше угол, и чем он острее - тем меньше сила.
Но в любом случае, энергия и работа определяются высотой, на которую следует поднять этот груз.

>Внезапно работа зависит от длины


> Хотя отсюда не очевидна ситуация с работой и энергией.


Да я уже третий раз тебе повторяю о потенциальной энергии mgh - в глаза не долбись.

Работа силы тяжести при перемещении тела вдоль наклонной плоскости из точки N в точку M1 и M2
будет равна силе тяжести, умноженной на высоту наклонной плоскости - и не зависит от угла наклона плоскости:
A = Ph, где P - вес, он равен mg, h - высота. Пикрелейтед.
Энергия сообщенная телу для подъема на высоту h равна Ep = mgh = Ph; P = mg;
где m - масса тела, g - ускорение свободного падения, h - высота, P - вес, давление.

>осталось разобраться что такое вес и масса не свихнувшись


Вес P - это сила. Вес равен mg. Поэтому, масса измеряется в килограммах, а вес - в Ньютонах.
А на наклонной плоскости сила давящая на плоскость равна - mg×cos(α).
А вот давление же - зависит от площади.
Давление p - равно силе F (или весу P), разделённой(-ому) на площадь S. p = P/S;

>Вот я это узнать пытаюсь. А не геометрические модели, какие-то соотношения. Это всё вторичные вещи. Почему саму суть нигде не пишут?


Так ты же толком сформулировать не можешь то, что же ты хочешь.
Я так до сих пор и не понял, что же ты там узнать пытаешься.
Про плоскость и рычаг я тебе уже пояснил. Вопросы есть?

>Алсо гляньте вот этот пример.


Зачем мне ещё раз смотреть 7-ми минутное видео, которое я уже несколько раз смотрел,
если закон рычага описывается одной лишь формулой, которую достаточно понять и всё.

>Мне кажется они в видео сделали ошибку.


Ну так укажи её, мне что её искать ещё там, внутри видео, всё просчитывая?!!
И ваще тебе не похуй, запили видос лучше и вбрось сюда в вебм-ке, если такой умный дохуя.

Если что-то непонятно - то спрашивай что конкретно.
arch[1]5 Кб, 258x152
#54 #413544
>>413530

>Вот в этом проблема.


Я тебе уже пояснил чем наклонная плоскость смахивает на рычах, и даже нарисовал таки-рычаг из этой наклонной плоскости.
Ты что ещё не понял что-ли? Что ты не понял там?

>Ведь не длина делает работу, а сила (вес).


Работу по подъему груза делает разница сил, которая определяется силой f,
приложенной для подъема груза, и силой, которая тянет груз вниз - mg×sin(α);
Исходя из первой картинки >>413505
И величина этой силы прямо пропорциональна значению угла в градусах,
т. е. чем этот угол больше, и стремится к прямому, тем больше сила.
Чем меньше угол, и чем он острее - тем меньше сила.
Но в любом случае, энергия и работа определяются высотой, на которую следует поднять этот груз.

>Внезапно работа зависит от длины


> Хотя отсюда не очевидна ситуация с работой и энергией.


Да я уже третий раз тебе повторяю о потенциальной энергии mgh - в глаза не долбись.

Работа силы тяжести при перемещении тела вдоль наклонной плоскости из точки N в точку M1 и M2
будет равна силе тяжести, умноженной на высоту наклонной плоскости - и не зависит от угла наклона плоскости:
A = Ph, где P - вес, он равен mg, h - высота. Пикрелейтед.
Энергия сообщенная телу для подъема на высоту h равна Ep = mgh = Ph; P = mg;
где m - масса тела, g - ускорение свободного падения, h - высота, P - вес, давление.

>осталось разобраться что такое вес и масса не свихнувшись


Вес P - это сила. Вес равен mg. Поэтому, масса измеряется в килограммах, а вес - в Ньютонах.
А на наклонной плоскости сила давящая на плоскость равна - mg×cos(α).
А вот давление же - зависит от площади.
Давление p - равно силе F (или весу P), разделённой(-ому) на площадь S. p = P/S;

>Вот я это узнать пытаюсь. А не геометрические модели, какие-то соотношения. Это всё вторичные вещи. Почему саму суть нигде не пишут?


Так ты же толком сформулировать не можешь то, что же ты хочешь.
Я так до сих пор и не понял, что же ты там узнать пытаешься.
Про плоскость и рычаг я тебе уже пояснил. Вопросы есть?

>Алсо гляньте вот этот пример.


Зачем мне ещё раз смотреть 7-ми минутное видео, которое я уже несколько раз смотрел,
если закон рычага описывается одной лишь формулой, которую достаточно понять и всё.

>Мне кажется они в видео сделали ошибку.


Ну так укажи её, мне что её искать ещё там, внутри видео, всё просчитывая?!!
И ваще тебе не похуй, запили видос лучше и вбрось сюда в вебм-ке, если такой умный дохуя.

Если что-то непонятно - то спрашивай что конкретно.
#55 #413552
Я пришел поговорить с аноном о голографическом принципе. Где находится та самая плоскость на которой описывается физика? Или же любая поверхность описывает то, что внутри нее? Или как вообще? И почему(если я правильно понял) этот принцип говорит о том, что данный в одной области пространства описывают любую другую область пусть и очень далекую?
#56 #413553
>>413544
Да понял я всё, я говорю меня не формулы интересовали и принцип работы.

>Вес P - это сила. Вес равен mg. Поэтому, масса измеряется в килограммах, а вес - в Ньютонах.


Вот у меня на столе лежит ложка масса которой 100г, я её слегка приподнимаю от стола и весы которыми я поднимаю показывают 70г. Это что уменьшилось? Вес? Так вес это mg - тут ниче не меняется. Я понимаю что 30г всё ещё давят на стол, но как это называется?

>Ну так укажи её, мне что её искать ещё там, внутри видео, всё просчитывая?!!


я же тайминг указал и скрин сделал, там минута всего, хотя пох уже, энивей спасибо за усидчивость.
#57 #413554
>>413553

>Вот у меня на столе лежит ложка масса которой 100г,


>я её слегка приподнимаю от стола и весы которыми я поднимаю показывают 70г.


>Это что уменьшилось? Вес? Так вес это mg - тут ниче не меняется.


>Я понимаю что 30г всё ещё давят на стол, но как это называется?


Вес - это сила, измеряется в Ньютонах.
P = m×g, где m - масса, g - ускорение свободного падения.
Давай проведём проверку единиц:
[кг]×[м/с2] = [кг×м/с2] - эта единица измерения эквивалентна Ньютонам.
1 Н = 1 кг·м/с2 https://ru.wikipedia.org/wiki/Ньютон_(единица_измерения)
Весы измеряют именно силу, которая давит. Когда ты трогаешь груз,
ты создаешь противодействующую силу. Так вот, равнодействующая этих сил делённая на g даёт массу, которую показывают весы. Если P = m×g, то m = P/g;

>давят


А давит скорее не вес, а давление. Сила давления (как и любая другая сила, в том числе и вес) - измеряется в ньютонах.
Давление же измеряется в паскалях. Паскаль (1 Па) - это такое давление, которое производит
сила давления в 1 Н, будучи приложенной к поверхности площадью 1 м2: 1 Па = 1 Н/м2.
#58 #413621
>>413406
Разница в том, ЧТО понимать под сигналом и под информацией.
Например, на приёмник подаётся повышенное напряжение. На экране появляется "1". Оператор (лучше даже не живой человек, а робот) увидел единицу на экране и запускает ракету по Америке. Долбоёбы скажут: а-за-за, но ведь повышенное напряжение - это аналоговый сигнал. На самом деле нет, вернее да, но важно другое. Если принимать за информацию "1", то да, она была получена аналоговым сигналом. Но оператору единичка до жопы. Ему важно запускать ракету или нет. То есть информация - это команда на запуск ракеты. А эту информацию он получил из цифровой единички, то есть в цифровом виде. По сути сигнала было два, и информации две:
-Аналоговый (напряжение), из которого мы получили информацию, и стали оперировать ей как цифровым сигналом "1"
-Цифровой сигнал "1", который дал информацию, что надо запускать ракету.
#59 #413624
А квантовый комп сможет запароленные архивы и шифрованные харды вскрывать не хуже терморектального метода?
#60 #413631
>>413624
Лично я знаю, что квантовое ускорение в квантовых ускорителях,
если квантовый компьютер позволит оперировать кубитами быстрее чем битами -
позволит использовать квантовые алгоритмы: https://ru.wikipedia.org/wiki/Квантовый_алгоритм
и https://ru.wikipedia.org/wiki/Алгоритм_Шора
А эти алгоритмы могут вскрывать потоковое RSA и Diffie-Hellman'a чуть ли не
в режиме реального времени, ну и дискретное логарифмирование на эллиптической кривой.
Это всё обратные задачи экспоненциальной сложности, входящие в Класс NP,
если конечно - они решаются брутфорсом на обычных машинах.
Квантовые алгоритмы позволили бы куда быстрее решить эти задачи, при быстром оперировании
кубитами в процессе флуктуаций этих всяких спинов элементарных частиц в какой-нибудь кварк-глюонной плазме внутри разогретого термоядерным синтезом фузора, так что там не просто биты бы мелькали, а кубиты, и ещё и быстро словно перевёртыши, лол.
Но ИМХО, такие устройства скорее напоминали бы узкоспециализированные ASIC'и
или FPGA, предназначенные сугубо лишь для быстрого взлома стандартной крипты.
В то же время массовое внедрение квантовых ускорителей породило бы квантовое шифрование,
которое ещё сложнее взломать, если конечно квантовые вычисления
не позволят окончательно доказать равенство классов P и NP.
Тогда вообще был бы полиномиальный крутяк во всей своей полноте и абсолютная открытость.
И конечно же добавлю, что скорее всего - это всё уже есть!..
13989676584284102 Кб, 496x586
#61 #413638
Пикрилейтед.
Почему дед видит именно собаку? Почему пацан видит именно собаку? Отличается ли реальность деда от реальности пацана? Почему они видят именно собаку, а не бегемота допустим или же странную живую фигуру на четырёх осях?
Схуяли я вижу собаку на картинке, и деда, и пацана? Почему именно в моём понимании дедэто дед, пацана это пацан, а собака это собака? Это видит мозг или же япльзуюсь чужой информацией для определения существ на этих картинках*

Таблетки принимал сегодня.
#62 #413641
>>413638
Гугли ассоциативная память на рекуррентных нейронных сетях,
и распознавание образов посредством свёрточных нейросетей.
Алсо, наверни когнитивистику.
Ояебу39 Кб, 800x600
#63 #413645
>>413369
Сам призадумался над первым вопросом. Смерть это как сила притяжения, закон сохранения энергии, она просто есть и ее не быть не может, если рассматривать ее с точки зрения эволюции. Не было бы смерти не было бы эволюции, которую мы понимаем, не было бы и жизни.

У самого есть вопрос, на который вероятно не будет ответа.
Вопрос о динамике движения в газах и жидкостях.
Есть куб со стороной А, который движется в сторону указанную на картинке со скоростью V. На лицевой и противоположной к движению стороне установлены барометры ровно по середине заподлицо к поверхности. Также есть контрольный барометр, который показывает давление P.
Вопрос где будет больше разница давлений между контрольным и 2 или между контрольным и 3?
Будет ли соотношение этих разниц одинаковой при изменении скорости?
#64 #413648
>>413552
таки надеюсь, что найдется анон, который мне подскажет
#65 #413649
>>413645
Я не эксперт, но мне кажется при фиксированной скорости давление будет одинаково везде(если в космосе-вакууме без внешнего гравитационного поля, если же на земле, то на дно будет больше, на свод меньше, на остальные боковые стороны постепенное изменение: наверху малое, по середине средние, внизу максимальное), т.к. по СТО и ОТО движение при постоянной скорости ничем не отличается от состояния покоя, а вот при ускорении(в том же самом направлении что и v на твоем рисунке), скорее всего сильнее будет давить на боковую сторону где написано куб, и меньше где написано v со стрелкой, на 2 оставшихся боковых, свода и низа будет постепеное изменение: в самой левой стороне минимальное, в середине средние, в правом максимальное. Плюс те же самые добавочные сверху-вниз из гравитации если на Земле.
Но все что я описал, это скорее всего в случае жидкости, с газами наверное будет по другому и скорее всего везде будет одинаковое давление в случае газов. Но опять-таки повторюсь я не эксперт в этом вопросе.
#66 #413650
>>413648
>>413552
Погугли про теорему Нётер и изотропии и однородности пространства.
#67 #413651
>>413650
Я тупой, мне бы "на пальцах" объяснить
#68 #413652
>>413650
Я погуглил и не понял какое это отношение имеет к моему вопросу. Сжалься надо мной, анон, объясни.
#69 #413654
>>413552

>голографическом принцип


Это гипотеза же.
Но на тебе всё же стереографическую проекцию: https://www.youtube.com/watch?v=LnlhwE-G8zU
И ещё немного покушать принёс:
На первое для тебя будет чуток квантовой теории поля в пространстве-времени анти-де Ситтера.
На второе наверни - конформной теории поля под соусом конформной геометрии.
А затем щедро запей всё это AdS/CFT-соответствием.
#70 #413655
Господа, кто-нибудь может рассказать мне про байесовские сети в общем плане? Или хотя бы подсказать, по каким книгам лучше всего копать.
Дипломник дал несколько источников, из которых я склеил свой диплом, но проблема в том, что я практик, а не теоретик, и байесовские сети для меня, как судоводителя, будут иметь в работе чуть меньшее, чем никакое значение.
Но после разговора с научруком, я понял, что тему эту мне все же придется хоть как-то разгрызть.

Что понял в общем плане, вроде как этот метод позволяет чуть проще высчитывать вероятности каких-то событий при сложных вероятностных деревьях с кучей разных факторов, зависящих друг от друга.
С теорией вероятности у меня все не очень хорошо по причине того, что проходилось все давно, так что не кидайтесь пожалуйста.
#71 #413663
>>413552
Вся эта фигня с голографическим принципом, демагогия. Это непроверяемая фигня, первое. Второе пространство вообще существует? большой вопрос, есть законы физики, где фигурирует всегда 4-вектор, и само пространство просто следствие этих законов. как ты узнаешь где находится кошка? ну фотоны передают тебе эту информацию, фотоны это электромагнитное взаимодействие, у которого в параметрах есть 4 цифры, которые по каким-то законам меняются. То, что тебе кажется пространством, лишь субъективное ощущение изменения формулы взаимодействия частиц. Говорить о конкретном "месте" вообще нет смысла, я могу напридумывать миллион разных голографических абсолютно точных физических законов, банально применив к имеющимся какие-то преобразования координат. И после начну затирать, что мы все на четыреxмерной сфере живем, и всякое такое. Просто наш мозг "придумал" представление в виде ощущения пространства, а длину волны в виде цвета. Можешь представить что было бы наоборот? Расстояние ты бы видел цветом, а цвет расстоянием ощущал. да вполне, упорись ЛСД и поймешь.
#72 #413666
Я правильно понимаю, некоторые константы в физике и не только-это что-то вроде костылей? Вот был какой-то записанный закон, но потом, через пару годиков, когда появились возможности или силы его проверить, и оказалось, что вычисления немного отличаются от того, что должно получиться, но тк закон в целом нормальный-просто дописывают недостающую циферку, чтоб правильно было.
Так чтоли?
#73 #413667
>>413446
Крутящееся кольцо нужных размеров и скоростей сделай и не выебуйся.
#74 #413668
>>413663
Так вроде как говорят, что гипотеза вполне проверяема. И ты так и не ответило на те вопросы, которые я задавал.
#75 #413669
>>413668
*не ответил
самофикс
#76 #413670
>>413668
Какой вопрос то. Как ответить на вопрос человеку, который сам не понял своего вопроса?
И нихуя это не проверяемо. они лишь могут проврить, можно ли всё свести к более простому описанию, если действовать на голографической сфере (фактически просто преобразуют законы). Это всё демагогия, ибо как я говорил, пространства вообще нет. Просто проверяют как можно "упаковывать" энтропию.
#77 #413688
>>413670
Я натыкался на что-то вроде "если гипотеза подтвердится, это будет означать, что информация содержащаяся в одной области пространства, будет содержать инфомрацию и о всех других областях"... то ли я неправильно понял... то ли...
#78 #413697
>>413655

>Господа, кто-нибудь может рассказать мне про байесовские сети в общем плане?


Можешь навернуть определение из википедии: https://ru.wikipedia.org/wiki/Байесовская_сеть
Формально, байесовская сеть — это направленный ациклический граф, каждой вершине
которого соответствует случайная переменная, а дуги графа кодируют отношения условной
независимости между этими переменными. Вершины могут представлять переменные любых типов,
быть взвешенными параметрами, скрытыми переменными или гипотезами.
Существуют эффективные методы, которые используются для вычислений и обучения байесовских сетей.

>Или хотя бы подсказать, по каким книгам лучше всего копать.


Вот тут посмотри:
http://gen.lib.rus.ec/search.php?req=байесовские+сети
http://gen.lib.rus.ec/search.php?req=Bayesian+network

>я практик, а не теоретик


Главное понять саму суть, и для чего они вообще надо.

>Что понял в общем плане, вроде как этот метод позволяет чуть проще высчитывать вероятности


>каких-то событий при сложных вероятностных деревьях с кучей разных факторов, зависящих друг от друга.


Поскольку они могут обучаться, я полагаю, что напротив они, как и нейронные сети,
по вероятностям могут классифицировать, упорядочивать и выводить закономерности, довольно точно,
или приблизительно точно.

Например, байесовская сеть может быть использована для вычисления вероятности того,
чем болен пациент по наличию или отсутствию ряда симптомов, основываясь на данных
о зависимости между симптомами и болезнями.
#78 #413697
>>413655

>Господа, кто-нибудь может рассказать мне про байесовские сети в общем плане?


Можешь навернуть определение из википедии: https://ru.wikipedia.org/wiki/Байесовская_сеть
Формально, байесовская сеть — это направленный ациклический граф, каждой вершине
которого соответствует случайная переменная, а дуги графа кодируют отношения условной
независимости между этими переменными. Вершины могут представлять переменные любых типов,
быть взвешенными параметрами, скрытыми переменными или гипотезами.
Существуют эффективные методы, которые используются для вычислений и обучения байесовских сетей.

>Или хотя бы подсказать, по каким книгам лучше всего копать.


Вот тут посмотри:
http://gen.lib.rus.ec/search.php?req=байесовские+сети
http://gen.lib.rus.ec/search.php?req=Bayesian+network

>я практик, а не теоретик


Главное понять саму суть, и для чего они вообще надо.

>Что понял в общем плане, вроде как этот метод позволяет чуть проще высчитывать вероятности


>каких-то событий при сложных вероятностных деревьях с кучей разных факторов, зависящих друг от друга.


Поскольку они могут обучаться, я полагаю, что напротив они, как и нейронные сети,
по вероятностям могут классифицировать, упорядочивать и выводить закономерности, довольно точно,
или приблизительно точно.

Например, байесовская сеть может быть использована для вычисления вероятности того,
чем болен пациент по наличию или отсутствию ряда симптомов, основываясь на данных
о зависимости между симптомами и болезнями.
#79 #413698
>>413326 (OP)
Как работает система хранения данных в супер компьютерах?
#80 #413708
>>413688
правильно.
#81 #413721
>>413715

>1 пик


Это творчество душевнобольных?
#82 #413722
Какой толк от гравитационных волн для человечества?
#83 #413733
>>413698
Держи цифровой документ с мануалом спецификаций стандартов: http://2014.nscf.ru/TesisAll/0_NSCF_Plenar/17_184_TutlyaevaEO.pdf
В основном это массивы RAID параллельные и последовательные с LBA-адресацией,
поддерживающие резервное копирование и отказоустойчивость в режиме реального времени.
https://ru.wikipedia.org/wiki/RAID
#84 #413740
Что будет, если засунуть сперму паука в пизду женщины, ну это, как там его, оплодотворить яйцеклетку спермой паука. Что-нибудь родиться? человек-паук))) но я серьезно. И проводятся ли такие опыты?
#85 #413743
>>413740
Не сможет оплодотворить - у пауков все совсем не так. А вот обезьянья сперма может оплодотворить, но потомства не будет - не будет образовываться зигота.
1171149515501c7dc22co614 Кб, 2556x1534
#86 #413745
Попался мне на глаза на днях фильмец про плоскую землю. Фантазия конечно, но один их аргумент я не смог объяснить.

Речь идет о том, что плоскофаги оспаривают тот факт, что округлость Земли можно легко проверить опытным путем, наблюдая как корабль скрывается за линией горизонта, то бишь за кривизной земли.

Опровергают они это тем, что если якобы вооружить свой глаз каким-нибудь телескопом или камерой с мощным зумом, то исчезнувший за горизонтом корабль можно снова выудить в поле зрения при помощи зума. И на ютюбе действительно есть такие видео и их даже немало. Как это адекватно объяснить? Ведь если бы корабль действительно уходил за кривизну Земли, никаким телескопом его уже невозможно было бы увидеть.

https://www.youtube.com/watch?v=kHvkyfE2eQc
https://www.youtube.com/watch?v=m-pbyJSSoxA
https://www.youtube.com/watch?v=EkuzlaLyTXM
#87 #413748
>>413745
Из-за линз.
1024px-Superiormirage[1]76 Кб, 1024x640
#88 #413755
>>413745
Тоже видел то видео - не могу найти.

>корабль скрывается за линией горизонта


>если якобы вооружить свой глаз каким-нибудь телескопом или камерой с мощным зумом, то исчезнувший за горизонтом корабль можно снова выудить в поле зрения при помощи зума


>есть такие видео и их даже немало


>Как это адекватно объяснить?


Пикрелейтед - за горизонтом движется корабль нормальных размеров. При специфическом состоянии атмосферы его отражение над горизонтом кажется гигантским.

Это либо мираж, который называется Фантом дальнего видения.
Появляется мираж дальнего видения тогда, когда земная поверхность нагревает воздушные массы, после чего они уходят вверх и охлаждаются. Если наверху над слоем холодных воздушных масс в силу тех или иных причин окажется более тёплый (например, его сюда занесли ветра с юга) и при этом сильно разреженный слой, а разница температуры между ними окажется довольно большой, то произойдёт рефракция. Световые лучи, которые отражаются от предметов, что расположены на земной поверхности, сделают дугу и уйдут обратно вниз, но уже не к своему непосредственному источнику. Они оказываются в десятках, а иногда и в сотнях километрах от него.

Одним из ярких примеров таких фантомов является Летучий Голландец.

Либо же, скорее всего - это, Верхний мираж.
Глянь вот тут внутри, короче: https://ru.wikipedia.org/wiki/Мираж
#89 #413776
Антуан. Нахуя люди общаются? Почему людям доставляет это удовольствие? Накидайте статей/журналов/книг, а то я даже не знаю как запрос в гугле сформулировать, чтобы найти ответ
#90 #413778
>>413776
https://ru.wikipedia.org/wiki/Общение
Общение — сложный многоплановый процесс ... включающий в себя как минимум три различных процесса:
коммуникацию (обмен информацией),
интеракцию (обмен действиями)
и социальную перцепцию (восприятие и понимание партнера).
Можешь рассмотреть эти составляющие поподробнее.
#91 #413799
>>413776
С. Савельев
выв267 Кб, 1038x594
#92 #413814
анон, поясни за прецессию гироскопа. я вроде находил информацию, но каждый раз какой-то момент да не понимаю. вот например. показывают "торк" все дела, но я думал что речь идёт о оси момента. а потом они, хуяк и складывают вектора. как они блять определили что "торк" идёт к нам а не от нас? я не понимаю. можешь обьяснить?
#94 #413817
#95 #413820
Привет, ананасы! Не знаю в какой раздел это спросить, поэтому спрошу сюда. Наверняка же здесь есть господа, шарящие в инет технологиях. Интересует вопрос шифрования данных и анонимности в сети. Особенно луковая и чесночная маршрутизация. Где можно детально изучить этот вопрос? На первых страницах гугла одна вода, общий принцип без деталей. А мне именно детали и нужны! Посоветуйте подробных статей или видеолекций каких. Желательно на русском.
#96 #413821
>>413815
ты вопрос прочитай, гироскопы я блять и до этого видел.
1 видео "смотрите я собрал гироскоп"
2 видео "есть такая штука как гироскопы, у них есть прецессия"
это именно то что я спрашивал.
#97 #413823
>>413821
вбил в гугле прецессия гироскопа. первая же ссылка (после вики) http://learning.itsoft.ru/coding/first/lab04/index.html - в ней видно все с формулами и прочим счастьем. че такое торк я на твоей фене не понял. да и картинку твою не понял. как и не понял какие ты там вектора складываешь. понял только что тебе не понятно, какого хуя гироскоп поворачивается, когда ему на бок вниз давят. в статье все с картинками, с векторами(которые там и не складываются), с зависимостью этих векторов друг от друга. вопросы по статье есть?
#98 #413824
>>413820
Чесночная маршрутизация (англ. «Garlic Routing») — это технология анонимного,
зашифрованного обмена информацией через компьютерную сеть,
используемая в анонимной сети I2P.
Чесночная технология является расширением луковой маршрутизации, используемой в проекте Tor.

Чесночная технология, используя многослойное шифрование, позволяет единственному сообщению (так называемому «чесноку»)
содержать в себе множество «зубчиков» — полностью сформированных сообщений рядом с инструкциями для их доставки.
В один «чеснок» в момент его формирования перед отправкой закладываются множество «зубчиков», являющихся зашифрованными
сообщениями как нашего узла, так и чужими — транзитными. Является ли тот или иной «зубчик» в «чесноке» нашим сообщением
или это чужое транзитное сообщение, которое просто проходит через нас, знает только тот, кто создал «чеснок»,
никто иной узнать эту информацию не может. Чесночная технология применяется тогда, когда нужно отправить зашифрованное
сообщение через промежуточные узлы, у которых не должно быть доступа к этой информации.
https://ru.wikipedia.org/wiki/Чесночная_маршрутизация

Луковая маршрутизация (англ. Onion routing) — это технология анонимного обмена информацией через компьютерную сеть.
Сообщения неоднократно шифруются и потом отсылаются через несколько сетевых узлов, называемых луковыми маршрутизаторами.
Каждый маршрутизатор удаляет слой шифрования, чтобы открыть трассировочные инструкции и отослать сообщения на следующий
маршрутизатор, где все повторяется. Таким образом, промежуточные узлы не знают источник, пункт назначения и содержание сообщения.
https://ru.wikipedia.org/wiki/Луковая_маршрутизация

Луковая маршрутизация работает в соответствии с принципом смешанных соединений Чаума: сообщения передаются из источника к месту
назначения через последовательность прокси («луковых маршрутизаторов»), которые перенаправляют сообщение в непредсказуемом направлении.
Чтобы избежать «прослушивания» сообщений злоумышленником, между маршрутизаторами они передаются в зашифрованном виде.

Как правило используются асимметричное шифрование, RSA, DH, El-Gamal, или даже ECC, и цифровая подпись - например ECDSA.

TOR и I2P - это просто программное обеспечение, и система прокси-серверов или VPN-тоннелей, реализующие эти технологии.
#98 #413824
>>413820
Чесночная маршрутизация (англ. «Garlic Routing») — это технология анонимного,
зашифрованного обмена информацией через компьютерную сеть,
используемая в анонимной сети I2P.
Чесночная технология является расширением луковой маршрутизации, используемой в проекте Tor.

Чесночная технология, используя многослойное шифрование, позволяет единственному сообщению (так называемому «чесноку»)
содержать в себе множество «зубчиков» — полностью сформированных сообщений рядом с инструкциями для их доставки.
В один «чеснок» в момент его формирования перед отправкой закладываются множество «зубчиков», являющихся зашифрованными
сообщениями как нашего узла, так и чужими — транзитными. Является ли тот или иной «зубчик» в «чесноке» нашим сообщением
или это чужое транзитное сообщение, которое просто проходит через нас, знает только тот, кто создал «чеснок»,
никто иной узнать эту информацию не может. Чесночная технология применяется тогда, когда нужно отправить зашифрованное
сообщение через промежуточные узлы, у которых не должно быть доступа к этой информации.
https://ru.wikipedia.org/wiki/Чесночная_маршрутизация

Луковая маршрутизация (англ. Onion routing) — это технология анонимного обмена информацией через компьютерную сеть.
Сообщения неоднократно шифруются и потом отсылаются через несколько сетевых узлов, называемых луковыми маршрутизаторами.
Каждый маршрутизатор удаляет слой шифрования, чтобы открыть трассировочные инструкции и отослать сообщения на следующий
маршрутизатор, где все повторяется. Таким образом, промежуточные узлы не знают источник, пункт назначения и содержание сообщения.
https://ru.wikipedia.org/wiki/Луковая_маршрутизация

Луковая маршрутизация работает в соответствии с принципом смешанных соединений Чаума: сообщения передаются из источника к месту
назначения через последовательность прокси («луковых маршрутизаторов»), которые перенаправляют сообщение в непредсказуемом направлении.
Чтобы избежать «прослушивания» сообщений злоумышленником, между маршрутизаторами они передаются в зашифрованном виде.

Как правило используются асимметричное шифрование, RSA, DH, El-Gamal, или даже ECC, и цифровая подпись - например ECDSA.

TOR и I2P - это просто программное обеспечение, и система прокси-серверов или VPN-тоннелей, реализующие эти технологии.
#99 #413825
>>413821

>ты вопрос прочитай, гироскопы я блять и до этого видел.


>>413823

>какого хуя гироскоп поворачивается, когда ему на бок вниз давят


Я ж видео для чего оставил здесь? Там суть прецессии и изложена.
Направление прецессии очевиднейшим образом зависит от того в какую сторону вращается гироскоп.
#100 #413826
>>413823
спасибо, посмотрю сейчас. "торк" переводится как "крутящий момент", но как я понял это не совсем то. это как-бы не сам момент а его ось, но я не понял почему они в некоторых обьяснениях на эту ось накидывают вектор.
#101 #413827
>>413823
прости, но там пиздец всё снова сложно, видимо я совсем недоразвит для этой темы. я думал это можно как-то на пальцах для аутистов-дегенератов обьяснить без математики, на наглядной анимации например. понимаю что чтото совсем очевидное упускаю
ввв289 Кб, 2136x1080
#102 #413829
Короче я правильно понял.
вот у нас вращается колесо с точками а и б которые показывают направление инерции, сбоку повесили нагрузку.

колесо наклоняется. и появляются а' б' которые показывают куда будут двигаться эти точки, но направление инерции не совпадает. получается что колесо стремится по инерции продолжить двигаться по тем векторам, и из за этого начинает прецессировать? я правильно понял суть? или хуету написал?
#103 #413831
>>413824
Да, спасибо, я это уже читал. Мне нужно на самом подробном уровне, с разбором алгоритмов и математической кухней.
#104 #413839
Даже объяснения винта архимеда нету нигде, простейшие механизмы ага. Какой-то заговор интернетовский.
#105 #413841
>>413839
Это к какому посту сарказм? Задавай вопрос. На то мы и тут.
#106 #413843
>>413839
Ну я начал изучать механику с простейших механизмов и принципов их работы. Выше по треду уже были рычаги всякие. Так вот я нигде не нашёл объяснения почему вода в архимедовом винте поднимается вверх. Пока сам это штуку не сделаешь и не увидишь - хрен чё поймешь.
#107 #413844
>>413831
Если интересуют принципы работы...
Тут глянь TOR: http://novainfo.ru/article/6541
Тут I2P: https://habrahabr.ru/post/152771/
А вообще, тебе бы в исходники влезть.
И TOR и I2P представляют из себя свободное и открытое программное обеспечение.
Там, внутри в мануалах всё это должно быть, причём более углубленно в описаниях стандартных модулей.
#108 #413845
>>413839

>Даже объяснения винта архимеда нету нигде


В гугле забанили что-ли? А то яндекс, порой банят, лол.
>>413843

>Так вот я нигде не нашёл объяснения почему вода в архимедовом винте поднимается вверх.


Потому, что он крутится.
#109 #413846
>>413845
И чё? На гифке вообще точка материальная вместо воды. Второй рисунок не соответствует реальности. Вот же жидкая, почему она по спирали обратно не стекает?)
#110 #413847
>>413846

>Вот вода же жидкая, почему она по спирали обратно не стекает?)

#111 #413848
>>413847
блин, сложно такие вещи обьяснять на самом деле, посмотри на ту правую картинку. представь что вода между винтом и трубой не может вытикать, потому-что охуенно всё плотно сделано. получается что при вращении винта вода лежит в местах, где в данный момент самая низкая для неё точка, тоесть винт и в ту и в другоую сторону выше её. но за счет вращения постоянно поднимает массу наверх. если непонятно- напиши, попробую найти инфу максимально простую.
#112 #413850
>>413844
нахуй такую инфу в торе и и2п постить? боязнь что НАУЧНОЕ ТАЙНОЕ ЛОББИ найдёт?
#113 #413851
>>413846
ну вода, как и шарик на гифке лежит под действием гравитации на нижней для неё точке. посмотри на стопкадр гифки, если вместо шарика налить воду, она ведь не сможет никуда деться, будет лежать в этой ложбине.
#114 #413852
>>413848
Да я разобрался уже. Просто везде пишут так, будто описывают работу сверла по дереву, типа крутишь и вода сама затекает вопреки гравитации. И дурацкие картиночки в одной проекции где уровень воды везде одинаковый.
#115 #413853
>>413846
>>413847
Потому что гидравлика.
#116 #413854
>>413853

>гидравлика


Нет, постойте. Ещё гидродинамику, гидромеханику, и гидростатику наложу сюда.
#117 #413855
>>413854
гравитация
Безымянный33 Кб, 593x319
#118 #413866
>>413326 (OP)
Оправдывайтесь петушки.
#119 #413869
>>413866
Сначала научись элементарной алгебре и штаны не забудь постирать.
#120 #413870
>>413866
Найс ты на дробь не умеешь делить, пятый класс то закончил?
#121 #413872
>>413870
Дебич дробь переворачивается, лал.
>>413869

>элементарной


>иррациональные числа


ясн
1496381238893042 Кб, 593x319
#122 #413880
>>413872
Дебич, когда ты r делишь на два, ты умножаешь на одну вторую, а не делишь. Оттуда все твои ошибки, лол.
#123 #413882
>>413880
Я тибя зотролел, озоза.
#124 #413906
Если молния предпочитает двигаться к заряду, а молния - это электрическое поле, то почему электрическое поле предпочитает двигаться к заряду?
#125 #413911
Анон, читаю книжку по СТО,помоги понять один момент.
Вот на рисунке 24 прием ретранслированного сигнала от ракеты а1 к а2 вроде как точка Р, а прием сигнала ракетой бета1 точка N. И по графику видно что N произошло раньше чем P c точки зрения альфацентриста.
Теперь другая ситуация, пусть ракета б движется относительно ракеты а, и в обоих отмечается время окончания какого-нибудь одинакового процесса, как на рис 24. Как написано, это событие произойдет в ракете а раньше, потому что она неподвижна с т.з. находящегося там. Но почему тогда на первом графике N произошло раньше чем P???
#126 #413921
ВНИМАНИЕ ЗАДАЧКА
из пункта А в пункт Б навстречу друг другу одновременно двинулись пучки света, с какой скоростью сближаются эти два пучка?
я видел подобную задачу с двумя ракетами летящими со скорость 60% от скорости света и замедляющимся временем, но это же хуйня собачья, насколько я понимаю
#127 #413928
>>413906
молния не является электрическим полем.
#128 #413929
>>413928
Орнул. А еще она не движется со скоростью света, да?
#129 #413934
>>413921
со скоростью света
#130 #413935
>>413929
Орнул. Молния со скоростью света- это что-то интересное.
#131 #413937
Что научные хуи думают про технологическую автоматизацию(а точнее безработицу, связанную с ней) и всякие ВР-утопии?

Части из них похер, они все стары и собираются один за другим двигать коней в близжайшие(не ебу как пишется это слово мде) десятилетия. Это понятно.

Но их место займут другие( кто, кстати? есть ли 'молодые' версии Хокингов-Саганов-Как?).
#132 #413941
>>413934
...умноженной на 2?
#133 #413942
>>413941
нет
#134 #413943
>>413942
почему?
#135 #413944
>>413943
1. можешь воспользоваться формулой для сложения скоростей
2. на пальцах: из постулатов сто, свет в инерциальной системе распространяется со скоростью с. переходим в систему связанную с первым пучком(система покоя для первого пучка), другой пучок находится в инерциальной системе связанной с первым, из постулата следует, что он распространяется со скоростью с.
#136 #413949
Почему на северном полюсе температура сейчас -1 С, а на южном -60 С? Как такое возможно, разве они не равноудалены от экватора?
#137 #413950
>>413949
как ось вращения земли расположена?
#138 #413951
>>413935
А что там тогда движется помимо медленного заряда?
#139 #413957
>>413944
Ничего что там в преобразованиях в знаменателях нули появляются? Хе-хе? >:-)
#140 #413959
>>413957
не появляются
#141 #413961
Какими физическими и химическими свойствами будет обладать атом полностью лишенный всех электронов?
#142 #413963
>>413944
я налажал. просто подставив в формулы я получил c. но связать с пучком систему нельзя, надо подумать, как правильно ответить. Извините.
#143 #413966
>>413961
Химические свойства таковы что это будет очень очень сильный окислитель, при Z<9 скорее всего сильнее даже фтора.
Физические хуй пойми, но скорее всего это будет газ, а жидкостью и твердым телом только при очень сильном давлении, и то после снятия давления сразу распад в газ. Очень сильный "удельный" положительный заряд. В условиях наличия простых атомов и молекул, скорее всего очень малое время жизни, т.к. очень быстро начнут отбирать чужие электроны.
#144 #413986
>>413326 (OP)
Поясните за квантовую запутанность. Вот говорят что с её помощью нельзя передавать информацию выше скорости света, потому что если мы насильно изменим состояние частицы на нужное, то запутанность пропадёт. А разве нельзя именно это и использовать? Вот у нас есть два запутанных электрона, мы измеряем спин первого и из этого сразу знаем спин второго. Если спин первого мы поменяет, то спин второго будет 50/50 уже. Начали мы измерять спин второго электрона раз в секунду, например. Он +1, значит у первого -1. Измерили два раза, тот же +1, пять раз, всё так же +1. Но тут кто-то берёт и меняет спин первого электрона, запутанность пропадает и спин второго становится рандомным. Разве вот эта последовательность +1 +1 +1 +1 +1 -1 +1 +1 -1 не может нести информацию? Разве нельзя принять за сигнал появление в последовательности -1? Если что я в этом не разбираюсь, просто прочитал вот это. https://geektimes.ru/post/288092/
#145 #413988
>>413986
А что за неё пояснять то, нельзя передавать инфу быстрее света, это следует из уравнений.
#146 #413989
>>413951
Там ионы летят, и выбивают из других атомов электроны по цепной реакции делая новые ионы. Обычный пробой в конденсаторе.
#147 #413990
>>413988
Это объяснение уровня нельзя потому что нельзя, а мне нужно узнать где в моих рассуждениях ошибка.
#148 #413991
>>413990
Летят два фотона, оба в суперпозиции A и В, когда ты измеряешь у одного фотона спин, то ты тоже становишься в суперпозиции "увидел A", "увидел B", твой друг на другом конце измерил тоже, и тоже в суперпозиции, только вот если вы с другом встретитесь когда-нибудь, или по скайпу позвоните, то будет подобная хрень.
Есть 4 варианта, вы оба говорите A A, A B, B A, B B. Так вот, если посчитать вероятность событий AB, и BA, то она 50 % и 50%, а вероятность события AA, и BB, выйдет 0.
Взаимодействие же на фотон тобой, ни к чему не приведет, твой друг не будет знать что ты там делаешь, для него последующие варианты вполне себе разрешены, и имеют свои вероятности.
#149 #413992
>>413991
Понятно, видимо автор статью какой-то хуйни написал.
Universe9,1 Мб, 542x292
#150 #413993
Это правильная модель вселенной? Не припомню, таких ульев из галактик. Да и вроде есть теория, что материя не бесконечна.
#151 #413995
>>413992
Да, хуйню. Но хомяки верят, а всех комментаторов которые его обоссали минусят.
#152 #413996
Наукачь помогите достать книгу.
квантовые аспекты функционирования биологических структур
я знаю у тебя есть.
#153 #413998
>>413986

>если мы насильно изменим состояние частицы на нужное, то запутанность пропадёт


Не обязательно.

>Начали мы измерять спин второго электрона раз в секунду, например. Он +1, значит у первого -1.


После этого все, попрощайся со своими двумя электронами и бери новые.
1494974925978839 Кб, 636x1048
#154 #413999
>>413326 (OP)
Может ли пикрелейтед родословное древо принадлежать неинвалиду? Притом, что показанные живыми родственники тоже неинвалиды? Картинка с форчана, её автор утверждал, что он русин из эндогамного рода.
#155 #414000
>>413998
Но ведь измерение не нарушает запутанность.
#156 #414001
>>414000
После измерения у частиц вполне понятное состояние, например, у одной из них +1, у другой -1. Где тут запутанность?
#157 #414013
>>413996
Ой бля, на тебе квантовую химию:
https://ru.wikipedia.org/wiki/Квантовая_химия
И замешай её с биохимией и нейрохимией.
#158 #414019
>>414001
То есть два раз измерить мы не можем? Почему-то вот это нигде не упоминается.
#159 #414034
>>414019
Это же очевидно. Было некое состояние "а" у частиц, стало состояние +1 у одной частицы и -1 у другой, которые явно не равны изначальному состоянию "a".
#160 #414035
>>414034

>Это же очевидно


Хехе, ну может для самих физиков и очевидно. Но всё таки научпоп статье это неплохо было бы указывать тоже, чтоб быдло тоже понимало.
#161 #414041
Ладно, наверно я непонятно спросил. Вот есть два "поезда", один состоит из ракет а1 и а2, второй движется равномерно относительно первого и состоит из ракет б1 и б2. Когда а1 и б1 поравнялись, на них запускаются часы и излучаются сигналы к а2 и б2, отражаются и возвращаются обратно. Так вот на графике, построенном в системе альфа, момент возврата на рисунке 26 в системе альфа обозначен точкой P, а в системе бета точкой N. Но ведь система бета движется относительно альфы, поэтому там время должно замедляться, почему же тогда N произошло раньше, если смотреть по графику?
#162 #414043
>>414041
N не произошло раньше, с чего ты взял, этот график статичный, а ты "двигаешь" по нему пальцем. У тебя время обозначено снизу, же. И блять нахуй эти графики, считай через интервалы, всё проще будет.
#163 #414052
>>414043
вобщет время вертикальная ось, Xa=0, почему снизу?
#164 #414079
>>414052
Тогда зачем подписями вводить в заблуждение... Короче не еби себе мозги с графиками, это бессмысленно. И второе, у тебя относильно первого поезда, всё медленее во втором, а во втором поезде медленее в первом, хехе...
#165 #414089
>>413963
так может тогда все же теория с замедлением времени имеет место? но как тогда это выглядит для постороннего наблюдателя?
#166 #414091
>>414089
Время впорядке, просто погрешности в вычислениях
#167 #414092
Почему человек чувствует боль? Я по сути спрашиваю, про кох-и я знаю. Вопрос в том, почему я, как Я, испытываю боль. Как это работает?
file458 Кб, 256x256
#168 #414098
Если я буду дуть на чашку с холодной водой, это замедлит её остываниенагревание? до комнатной температуры, или ускорит?
#169 #414100
>>414098
Ускорит наступление термодинамического равновесия.
#170 #414118
Сап, аноны, накидайте литературы, которая рассказывает про связь квантовой механики и работы мозга, плес, друг просит.
#171 #414141
Писал сегодня огэ по физике(да, школоло)
И меня смутил один вопрос. Имееются 2 ткани, шелковая и бархатная
Если покрасить их одинаковой краской, нужно определить у какой цвет будет насыщеннее
Во время экзамена не знал об этих тканях нихуя, но подумал что шелк тонкий, так и написал - он дохуя света пропустит, бархат будет насыщеннее
Дома смотрю в интернете - бархат будет как ебаный вантаблэк поглощать свет, тк свет будет от ворсинок многократно отражаться и поглощаться в конце концов. А шелк как гладкая поверхность отразит больше, соответственно насыщеннее будет.
И я думаю, а не зависит ли это от типа шелка? Он же бывает такой легкий, что прозрачный будет
И что, мне хуй отсосать? Аноны, рассудите, я точно неправ или при определенных условиях бархат будет насыщеннее?
#172 #414151
>>414141
Ебать у вас там вопросы.
#173 #414152
>>414031
Люминь - металл оче дешевый, по сути, только при производстве много энергии требует. Проблема в том, что бетон в любом случае дешевле будет. С пластиком то же самое.
#174 #414153
>>413999
Да легко. Генетика - тот еще китайский рандом.
#175 #414154
>>414151
>>414151
А если серьезно, подам я аппеляцию как придут результаты, смогу доказать что от вида шелка зависит насыщенность цвета и при определенных условиях я правильно описал?
#176 #414157
>>414154
Хуй ты что докажешь.
#177 #414159
>>414157
Ладно
А чисто с вашей точки зрения, я проебался или все таки доля правды в моих словах есть?
на оценку похуй, сдам и сдам, интересно как я сам физику понимаю, ибо планирую что она моим предметом и будет
#178 #414160
>>414159
С моей точки зрения, это вопрос про двух верблюдов, один из которых красный а другой налево.
#179 #414161
>>414160
Тогда вот еще один
2 калориметра, один с эфиром точеным, другой с водой дроченый. t1=25c, будет ли разница в показаниях термометров в калориметрах спустя какое-то время. Больше ничего не дали
Тоже задача из сегодняшнего экзамена
#180 #414162
>>414161
Пиздец у вас там.
Если следовать их омской логике, кефирный будет холоднее.
#181 #414163
>>414162
Это 2 вопроса из 26
остальные все нормальная, простая физика 7-9 класса
но вот 2 задания всегда на какую-нибудь залупу, и они письменные. И всегда дадут такие условия, что каждый к своему выводу придет
Так, а почему холоднее? Я просто с этими экзаменами сплю 3 часа в день, уже совсем не понимаю
#182 #414164
>>414163
Потому. Эфир быстрее испаряется, да и в воздухе его обычно меньше чем воды.
#183 #414165
>>414164
Короче из-за испарения
ладно, спасибо
image365 Кб, 400x517
#184 #414168
Так, кароч, проверьте моё понимание ОТО.
Летят навстречу друг другу по прямой два фотона. Между ними сидит наблюдатель.
В системе отсчёта связанной с одним из фотонов, скорость второго фотона всё равно будет с?
Но при этом скорость сближения фотонов для наблюдателя будет ?
Т.е. на часах наблюдателя пройдёт в два раза меньше времени до встречи фотонов, чем на часах у одного из фотонов?
#185 #414171
>>414091
а как тогда без погрешностей?
и какой будет их скорость между собой, если они будут выпущены в противоположные друг от друга стороны из одной точки?
#186 #414184
>>414171
Не слушай его. Он тебя траллит.
#187 #414185
>>414168
Нет. У фотонов нет часов.
#188 #414187
>>414168
У фотона, там пространство сжалось, для него вообще времени не пройдет.
#189 #414336
>>413487

>Никто не знает, существуют ли черные дыры.


Ученым впервые удалось сжать атом при помощи рентгеновского лазера
Сфокусировав рентгеновский луч в точке размером 100 нанометров, они добились эффекта, равного тому,
который мог бы получиться от фокусировки всего солнечного света в одной точке.
Оказавшись под таким лучом, атом йода начал терять все свои электроны, повысив плотность,
и превратился в нечто подобное черной дыре, затягивающей в себя электроны из соседних атомов углерода и водорода.
http://www.mk.ru/science/2017/06/01/amerikancy-sozdali-svoyu-chernuyu-dyru.html

В SLAC исследователям удалось сжать атом йода и получить эту материю на несколько фемтосекунд (1фс = 10-15 с).
Это всё - потому что испарение чёрных дыр и излучение Хокинга.

https://ru.wikipedia.org/wiki/Чёрная_дыра#.D0.A2.D0.B5.D1.80.D0.BC.D0.BE.D0.B4.D0.B8.D0.BD.D0.B0.D0.BC.D0.B8.D0.BA.D0.B0_.D0.B8_.D0.B8.D1.81.D0.BF.D0.B0.D1.80.D0.B5.D0.BD.D0.B8.D0.B5_.D1.87.D1.91.D1.80.D0.BD.D1.8B.D1.85_.D0.B4.D1.8B.D1.80
#190 #414348
>>414344
Если бы атом йода был просто ионизирован до состояния ядра, он бы мог в силу кулоновского электростатического притяжения электронов притянуть к себе лишь 53 электона, так как атомный номер у йода - 53.
Более того, если бы его ядро разлетелось оно разлетелось бы спонтанным делением с вылетом альфа-частиц.
Но если бы ядро разлетеслось даже на нуклоны и же на пи-мезоны, отдельные кварки и глюоны,
этого атома бы не существовало, вместо этого был бы водород, нейтроны, и экзотические атомы
как например пионий - т. е. положительно заряженный пи-мезон с электроном,
позитроний, мюоний или же было бы зарегистрировано спонтанное деление ядра на более массивные, нежели альфа-частицы осколки...
Однако речь идёт о чёрной дыре - затягивающей в себя электроны из соседних атомов углерода и водорода!
Малое время жизни как-бы намекает на то, что это была-таки — черная дыра.
В википедии написано следующее:
Скорость испарения чёрной дыры тем больше, чем меньше её размеры.
При этом интенсивность испарения нарастает лавинообразно, и заключительный этап эволюции носит характер взрыва,
например, чёрная дыра массой 1000 тонн испарится за время порядка 84 секунды, выделив энергию,
равную взрыву примерно десяти миллионов атомных бомб средней мощности.

Т. е. по мере уменьшения размеров — чёрная дыра испаряется всё быстрее и быстрее, излучая излучение Хокинга.

Также о наличие именно чёрной дыры говорит значение мощности одного лазерного импульса.
Это значение составляет до 1020 ватт на квадратный сантиметр!
https://lenta.ru/articles/2017/06/01/hole/
#191 #414352
>>413326 (OP)
Зачем заниматься наукой в 21 веке?
Наука, технологии нужны для взаимодействия с миром.
Но в бытовом плане все уже давно достигнутно, остается только задроченная тонкая глубокая хуйня, которой занимаются в дата-цетрах орды хомячков.
И вот я реально не понимаю. Информации все больше. Отделять зерна от плевел все сложнее.
Смысл что-то читать, изучать, если со временем вся эта информация превратится в обыденный мусор и будет уже другая, более удобная и лаконичная теория всего.
Если смотреть с глобальной точки зрения, то школьник-хуесос с мировозрением, построенным на дваче и маняме, недалеко ушел от хуя, который пытается читать статьи и во что-то даже вникать.
Ну шаришь ты сейчас в квантмехе, все вокруг быдло, а ты один дарьтаньян. Но смысл? Через 200 лет открывают спидфорс и твой квантмех идет нахуй.
Такой-то ангст, а точнее уныние накатывает.
Один хуесос с одним авторитетом спорит с другим хуесосм с другим авторитетом. И нахуя? Вы все сорта говна.
#192 #414356
>>414352
Это процесс познания, и он увлекательный и интересный. Плюс банальное любопытство. И да, теории всего быть не может.
#193 #414357
>>414356

>он увлекателен


Но в каких степенях? Познавать мир можно и через желтую прессу и второсортное кинцо, и это не будет как что-то плохим по сути.

>не может


ТЫСКОЗАЛ? Все научные хуи, а конкретно-физики и математики, с тобой не согласятся.
#195 #414360
>>414358

>арифметическая формалность


>теория всего


кек
#196 #414361
>>414360
А ты решил теорию всего без математики делать? И да первое что сделал Гёдель, доказал что любую математику можно свести в арифметику, а после уже доказывал для арифметики.

сразу видно, каникулы начались
#197 #414362
>>414361

>решил


Не я, а все научные хуи, это суть науки и мечта в стремлениях. Без этого наука не имеет смысла, это всего лишь ковыряние вилкой в какашках.

И при чем тут твоя арифметика, еще раз?
#198 #414363
>>414362
Первое, ты теорию всего в каком формализме будешь составлять?
Второе, конкретно ты какое отношение к науке имеешь? У тебя публикации есть может быть?
#199 #414365
>>414363
А че сразу я-то? Да и вряд ли кто-то может знать, иначе бы давно составили.

Прямое, я же человек. Для публикаций я слишком нищ.
#200 #414366
>>414352

>Но в бытовом плане все уже давно достигнутно


Некорректное утверждение вне зависимости от того что ты подразумеваешь под "бытовым планом".

>Смысл что-то читать, изучать, если со временем вся эта информация превратится в обыденный мусор и будет уже другая, более удобная и лаконичная теория всего.


Снова некорректное утверждение. В современной науке ни одна "следующая" теория не отменяет "предыдущей". Только дополняет и устанавливает рамки применимости, ну а при граничном переходе новая теория обязана сходиться к старой (и описывать наблюдаемый мир). Специализированные теории почти всегда удобней в использовании в их соответствущих областях применения. Поэтому,

>Через 200 лет открывают спидфорс и твой квантмех


продолжает использоваться там где и использовался.

>школьник-хуесос с мировозрением, построенным на дваче и маняме, недалеко ушел от хуя, который пытается читать статьи и во что-то даже вникать


Только в манямирке школьника-хуесоса.

Возвращаясь к изначальному вопросу,

>Зачем заниматься наукой в 21 веке?


Да низачем. Не хочешь, не надо. Никто не заставляет.
#201 #414368
>>414365
Сейчас попробую объяснить простым языком. Вот придумал Ньютон свои законы, но вопрос, а почему они такие? Появляется квантмех, где законы Ньютона лишь следствия, вопрос новый, почему уравнение именно такое?(плюс, почему такие константы и почему такие массы).
Теории всё растут и растут, но в любой момент, ты можешь спросить, "а почему уравнение именно такое?" и это будет новый вопрос, который заставит научных работников ломать голову и думать, каким уравнением объяснить константы в нём, или показать, что других быть не может, процесс бесконечный. Любая математическая формулировка будет неполной. А если полной, то противоречивой.
#202 #414370
>>414366

>некореектное


пруф

>снова неккоректние


>не отменяет


Вот тут ты знатно серенькнул. В истории науки полно ситуаций когда сначала был один хайп, затем все менялось, получался другой хайп и вообще все шло по пизде.
Космологическая модель, атомарные модели, релятивизм, от которого у всех рвало жопы(и до сих пор рвет), да дохуйя всего.
Никаких тебе дополненений и рамок, чистое напяливание гандона на ракету типа ну давайте же засунем куб в круглое отверствие, они же должны подходить 11!!!!

>манямир


Но ты тоже витаешь в манямире

>зачем


Личностное проекции совкового долбоеба как есть.
#203 #414371
>>414368

>квантмех


>классика


Пиздец нахуй, я каким дебилом говорю. Скройся нахуй, ученый
#204 #414372
>>414357
Хокинг еще раз передумал и заявил, что теорема Гёделя о неполноте, вполне возможно, указывает на принципиальную ошибку в его первоначальных рассуждениях. Он написал: «Некоторые люди будут очень разочарованы тем, что не существует окончательной теории, которую можно сформулировать в конечном числе пунктов. Я раньше тоже принадлежал к этому лагерю, но теперь изменил мнение... Теорема Гёделя гарантирует, что для математиков работа всегда останется. Я думаю, что М-теория сделает то же самое для физиков».
#205 #414373
>>414372
При чем тут Петухокинг?
#206 #414374
>>414373
А он не научный хуй?покормил зеленого
#207 #414375
>>414374
1 из неисчислимого кол-ва научных хуев что-то сказал
Дальше что?
#208 #414376
>>414372

> Я думаю, что М-теория сделает то же самое для физиков


для математиков. Физика - это экспериментальная наука, то что одно многообразие меньшей размерности вложено нетривиальным образом в другое, вообще не проверишь еще лет 300, если вообще проверишь.
#209 #414377
>>414376

>физика эксперементальная


Теоретическая и мета ссыт тебе в ебло.
#210 #414378
>>414370

>пруф


Пруфы на отсутствие чего-то (новой науки или технологий, в данном случае) не предоставляются.

>атомарные модели


Старые модели никогда не взлетали. Их учат в школе только как исторический очерк и пример логики развития. То что взлетело пользуется там где точность хватает. Даже ебаная химия с "орбиталями" все еще жива и воняет, хоть это и бред с точки зрения квантов.

>релятивизм


Говно мамонта под названием галилеевский релятивизм, до сих пор пользуется везде куда не глянь. Главное знать пределы применимости. Когда ОТО порвут чем-то более общим, на масштабах, где ОТО применима с нужной точностью все равно будут пользовать ОТО.

>Космологическая модель


С астрофизикой не все так однозначно, так как действительно рабочих моделей по сути еще пока нет. Вернее способы проверки достаточно ограничены.

>Но ты тоже витаешь в манямире


Зато я его могу проецировать на большее количество людей, чем школосос - свой.
#211 #414379
>>414378

>пруфы не предоставляются


слив? как-то хуево, тебя и слушать не можно, один хуй балабол без аргументации
#212 #414380
>>414352

>через 200 лет откроют спидфорс


О, наш уголок сидаба протек
на самом деле, без ученых то спидфорс не откроется. И тут не обязательно быть ученым на зарплате, многие открытия совершались самоучками или хотя бы теми, кто работал один в своей лаборатории. Как минимум чтобы предложить теорию, не обязательно сидеть в НИИ каком-нибудь
Есть знакомый, который придумал одну поправку к системе управления камеры для спутников всяких, в то время учился в вузе еще. И ничего, приняли и после этого забрали его к себе, поэтому не обязательно кучу лет сидеть в лаборатории
#213 #414381
>>414377
это уже математика по большей части.
#214 #414382
>>414381
это уже маневрирования по большей части
#215 #414383
>>414380
Не откроется. Я не говорю что она НИНУЖНА. Я говорю что она не нужна массам, коими ты и я являемся.
И с прогрессом хуй ты что откроешьразве что боженька не соизволит метнуть лично в тебя желтую молнию.
Сижу я. У меня есть чай и жевачка. Котелок для кипячения.
Вау, я аж смогу провести примитивнейшие эксперементы по термодинамике! Может даже сумею открыть что я долбоеб, ведь вместо переливания мочи из стакана в стакан я мог бы прочитать об этом уже-проделанном эксперементе в сети!
#216 #414385
>>414375

>Все научные хуи, а конкретно-физики и математики, с тобой не согласятся.



>1 из неисчислимого кол-ва научных хуев



Сам себя опроверг обоссал.
#217 #414386
>>414379
Наебнись чайником Рассела, довен, а потом кукарекай про сливы.
#218 #414387
>>414385

>опроверг


>коллективно-бессознательное


>новость о хуе который еще пару раз мнение свое поменяет


какой влажный манямир
#219 #414388
>>414386
ЧТо?
#220 #414389
>>414377

>мета


я сразу не заметил подвоха, у меня нет к тебе вопросов
#221 #414390
>>414383
Нынешняя физика не экспериментальная нихуя уже
это было в 19, начале 20 века
Сейчас экспериментами только проверяется, но не открывается что-то
Разве что бегать восьмерочкой не получится
#222 #414391
>>414390
физика принципиально - экспериментальная, потому что главный критерий - это эксперимент. За маняфантазии, которые укладываются математическую модель, премию никогда не дадут.
#223 #414394
>>414390
Твои маняфантазии никто не будет слушать если на практике они не работают. Поэтому ты обязан будешь так или иначе воплотить тот или иной эксперемент.

И не пизди ска, даже в процессе придумывания маняфантазии ты должен изучать, наблюдать, систематизировать и балбла объект, в этом суть методологии науки.

Ты не сможешь в комнатных условиях нафантзировать себе как работает сверхтонкие ребристые сплавы металлов с каким-либо веществом.
Чтобы чтобы понять и додумать ты должен проводить бесконечные эксперементы, блеа. Во все времена и тем более сейчас, и тем более в будущем, заниматься физикой-значит занимать опытами, будет.

Дохуя, если не большинство открытый было сделано ПРОСТО ПОЛУЧИЛОСЬ. Чувак сожрал особый полимер случайно-ууу дебил, но слушай, ты еще живой, нихуя себе, походу этот полимер можно человеку жрать! Так и запишем и вечером еще раз проверим...
#224 #414395
>>414390

>Нынешняя физика не экспериментальная нихуя уже


это не правда.
никаким другим образом кроме эксперимента например нельзя установить как будет вести себя материальное тело в нестандартных условиях, типо лягушки в сверхмощном магнитном поле.
ну только нормальные люди не с лягушками экспериментируют, а, например, с суперпарамагнитными телами неправильной формы в сверхмощных полях.
реальность такова, что вместо обсчета на суперкомпьютере выгоднее в ручную выпилить что нужно и датчиками замерить окружающие поля. это просто на порядок другой дешевле, потому что на определенного рода расчеты уходят в прямом смысле года для динамических систем.
#225 #414396
>>414395
Какие расчеты нахой если главный критерий это проверка опытом?
#226 #414397
>>414396
ты странный какой-то.
есть например пакеты ansys maxwell.
можешь посмотреть что они считают и с какой скоростью. после опыта еще бы неплохо модель для опыта какую-нибудь обсчитать.
а там итераций в определенных случаях ОЧЕ МНОГО надо проделать с одними и теми же входными данными.
другой вопрос, что чтобы получить модель ты уже должен провести эксперимент на колене, чтобы задать условия эксперимента в программе для расчета. чтобы потом уже посмотреть более подробно что там происходит и в какие моменты времени.
#227 #414398
>>414388
Ебать ты малограмотная школота. Пиздуй тогда в википедию про чайник читать.
#228 #414399
>>414398
При чем тут чайник, самопроецирущий малолетний дебил?
Сереня обосралсь и вместо того чтобы тихо ретироваться продолжать срать.
#229 #414400
>>414397
Тебе в любом случае нужно будет делать эксперементы, лалка. Как симуляция невесомости австронавтом в подвальных бассейнах.
#230 #414401
>>414399

>Ляпнул хуйню про ноуку


>Потребовал доказательства своей неправоты


>кококо-самопроецирущий малолетний дебил


Ебаные каникулы.
Хотя, судя по агресивности, про чайник таки нашел статью, уже прогресс. Дальше дело за малым, надо просто включить мозги.
1445611656hot-babe-pov-blowjob2,1 Мб, 460x690
#231 #414402
1) Чем отличается живое от неживого?
Вот есть клелка - она живая.
Вот есть набор молекул - он не живой.
Вот есть вирус - ну хуууй знает.
Вот есть вирионный агент - вообще хуй проссышь.
2) Чем определяются свойства материи, известной как элементы таблицы менделеева?
Почему ртуть - жидкая? Почему сера - жёлтая? Почему железо ржавеет целиком, а алюминий покрывает тонкой оксидной плёночкой и рад?
3) Как искать протезные фирмы РФ?
В России тоже есть инвалиды и есть даже те, кто делают протезы. Есть протезоделы уровня палка_с_гвоздями, а есть норм вообще, даже 3Д-печать. Где их искать, как нагуглить?
#232 #414403
>>414402
1) Четкой границы нет, как и нет четкой границы между космосом и атмосферой, холодно и горячо, мы сами придумываем что считать границей, какие признаки и насколько проявленные. Определений жизни очень много.
2) Это всё химические и физикохимические свойства, определяются они энергиями связей, межмолекулярными взаимодействиями, электронными переходами(цвет), кинетическими причинами(скорость коррозии), и т.д. Если бы у меня был очень очень мощный комп, можно было бы записать уравнение шредингера, для этого большого объема частиц, и рассчитать свойства, и цвет, и химические.
3)Не могу тебе сказать. но думаю что гугл знает ответ, так как любая фирма любит рекламу.
#233 #414404
>>414401
Забей, реально каникулы. Видимо не стал ЕГЭ по физике, не поступил, остался у себя в мухосране, решил самоутвердиться на двачах, а его еще и тут в говно макают.
tumblroprv0oeNmD1ug7p28o140089 Кб, 384x379
#234 #414406
>>414403

> 3)Не могу тебе сказать. но думаю что гугл знает ответ, так как любая фирма любит рекламу.


Да вот как-то не очень
#235 #414412
>>414401

>Некорректное утверждение вне зависимости от того что ты подразумеваешь под "бытовым планом".


>Пруфы на отсутствие чего-то (новой науки или технологий, в данном случае) не предоставляются.


Лол бля, обсерыш-семен все еще не может угомониться.
#236 #414414
>>414403

>четкой границы между космосо и атмосферой


Ой бля, лучше тебе не отвечать на вопросы, эксперт из 7б.
#237 #414415
>>414408

>маняфилософы


>четкая научная методология


...
#238 #414416
>>414414
Ну и где она? Просвети меня где заканчивается атмосфера, эксперт из 9а.
#239 #414417
>>414416
Как минимум на ум должна прийти мыслишка про выход через космическую скорость к орбиталям, как максимум-загуглить параметрические данные наса. Есть четкие диапазоны разделения блабла-сфер земли по разным характеристиками.
#240 #414418
>>414417
There is no firm boundary where outer space starts. However the Kármán line, at an altitude of 100 km (62 mi) above sea level, is conventionally used as the start of outer space in space treaties and for aerospace records keeping.

Как я понимаю, граница в разных науках проводится по-разному... Ах нет, у нас же только NASA есть, точно! А еще люди только в астрономией и астронавтикой занимаются, и только в NASA.
#241 #414419
>>414418
Как ловко ты сманеврировал!
#242 #414428
>>413326 (OP)
Есть такой продукт, как "морская капуста". Пишут, что он очень полезен, содержит много минералов и пр. Однако, как я узнал, в процессе приготовления его бланшируют (варят, проще говоря) на протяжении более получаса. По идее за это время все полезные вещества оттуда давно должны были уйти (точнее остаться в воде), а продают нам бессмысленную массу клетчатки, без какого бы то ни было существенного количества минералов. Я прав?
#243 #414437
>>413326 (OP)
Как в России выжить научному сотруднику?
Действительно ли важен Индекс Хирша?
#244 #414447
>>414428
Вываривается все органического происхождения, витамины тоесть, йод и прочие минералы неорганические и им насрать
#245 #414448
>>414402

>1) Чем отличается живое от неживого?


Зависит от того, что именно ты подразумеваешь под термином "жизнь".
Клетки выживают благодаря негэнтропийным процессам за счёт притока энергии,
тем самым продолжая существовать и даже делятся.
Но помимо негэнтропии есть ещё и экстропия, она свойственна многоклеточным
и другим организмам как сложным системам или даже суперсистемам, существующим в природе:
http://ru.science.wikia.com/wiki/Экстропия

>2) Чем определяются свойства материи, известной как элементы таблицы менделеева?


Химические свойства химических элементов а также реакционная способность -
определяются энергетическими уровнями электронов на электронных оболочках и их местоположением.

>Как искать протезные фирмы РФ?


Ключевые слова: Имплатация, протезирование, изобрели/создали/придумали протез,
можешь забить эти ключевые слова - в новости по гуглу, и обратиться прямиком в научно-исследовательские центры,
указанные там в статьях на новостных сайтах, при публикации инфы инновационных открытий.
#246 #414454
>>414400
так изначально речь об этом и шла, что чтобы что-то считать - надо поставить серию экспериментов, чтобы получить хотя бы приблизительную картину того что происходит.
#247 #414455
>>414402

>даже 3Д-печать


очевидно что нужно искать тех кто занимается печатью. но там вопрос денег и их желания.
штучные изделия у них стоят ГОРАЗДО ДОРОЖЕ чем серия.
#248 #414456
>>414454
Да, это взаимосвязанные активности, одно без другого бесполезно
#249 #414457
>>413326 (OP)
Скорость может расти бесконечно ведь? типа 99.9% скорости света, 99.99%, 99.999%, 99,9999...9 и так до бесконечности?
#250 #414458
>>414457
Ты только что парадокс Зенона.
#251 #414463
>>414457
Получится черная дыра, в итоге.
#252 #414493
>>414457
а объяснит кто-нибудь, почему скорость выше скорости света невозможна?
#253 #414496
>>414493
Это постулат. Тут увы нельзя ничего сказать.
24116600[1]55 Кб, 600x332
#254 #414499
>>414493
Световой конус, потому что.
#255 #414506
>>414499
типо поэтому вселенная и расширяется постоянно? со скоростью света?
#256 #414507
>>414506
быстрее
#257 #414509
>>414507
но ведь

>скорость выше скорости света невозможна

#258 #414511
>>414509
скорость расширения пространства это другое
#259 #414513
>>414509
Ты это вселенной объясни, что она оказывается НЕ МОЖЕТ. Люди так решили.
#260 #414540
>>414513
скорость расширения простарнства выше скорости света, ему уже ответили
#261 #414542
>>414511
>>414540
т.е. в теории можно построить двигатель, который сжимал бы пространство и ехал быстрее скорости света?
#262 #414543
>>414542
*расширял я имел ввиду
#263 #414547
>>414542
>>414543
Гугли варп-двигатель и пузырь Альбукерке
#264 #414559
>>414547

>варп-двигатель


сжимая пространство он укорачивает себе путь, правильно?
это как если сжать пружину, взяться ближе, и потом вернуть обратно?
#265 #414565
>>414559
Варп двигатель впереди сжимает пространство, а сзади расширяет, или наоборот.
Как работает и почему не знаю, я лишь знаю что есть такая штука как варп-двигатель и что она для того что бы путешествовать(не двигаться) быстрее скорости света. Поэтому отправил тебя гуглить
>>414547-кун
#266 #414567
Во время горения спички тепловая энергия переходит в электромагнитную? Ведь излучается свет, а свет это электромагнитная волна.
#267 #414573
>>414567
Электромагнитная в кинетическую. Плюс химические реакции это электромагнитные взаимодействия.
#268 #414575
>>414567
Только часть. Часть энергии уходит на образование химических связей новых соединений.
#269 #414585
Когда люди смогут поднимать тяжести в воздух, используя антигравитацию, аки египетские фараоны по РЕН-ТВ? Бозон Хиггса поможет в этом деле?
#270 #414586
>>414585
нет, не поможет
#271 #414590
Допустим человечество прилетело на другую планету, где есть жизнь.
Вопрос 1: Сможет ли человек есть пищу из инопланетных зверей и растений, имеющих ДНК из других нуклеотидов? Если все эти расщеплённые белки, нуклеиновые кислоты, полисахариды и т.д. будут иметь совершенно другое химическое строение, то сможет ли человеческий организм их усвоить?
Вопрос 2: Если на этой планете будет такая же атмосфера, как на Земле, но вирусы будут иметь другую РНК/ДНК, то насколько будет опасно человеку гулять по планете без скафандра? Если инопланетный вирус попадёт в клетку, то как он сможет встроиться в ДНК человека, ведь он имеет совершенно иное строение, чтобы потом на его основе с помощью факторов транскрипции создавалась мРНК?
#272 #414591
>>413369

>1) Для чего нужна смерть?


Ты неправильный вопрос задаёшь. Смерть нужна для того, чтобы организмы могли быстро эволюционировать и приспосабливаться путём естественного отбора. А вот для чего нужна жизнь - хороший вопрос.
#273 #414592
>>413411
длина волны и все, гугли "квалиа"
#274 #414595
>>414591
Зачем искать смысл, в том, в чем нет смысла.
Это как я буду подходить к каждому камню на улице, и справшиать "в чём твой смысл, зачем ты тут есть?"
#275 #414596
>>414567
Светится ионизированный в процессе химической реакции окисления (горения) газ,
состоящий из горящих продуктов пиролиза горючего состава и древесины,
а также ионизированных продуктов их реакции с кислородом.
#276 #414602
>>413326 (OP)
Поясните за Курсеру
https://ru.coursera.org/instructor/petersinger

Если я зарегаюсь на курс то тип обязательно надо будет домашнее задание выполнять? Я просто хочу видюшки позыркать а в свободном доступе их нет.
#278 #414608
>>414590
1)Нет
2)Смертельно опасно
CNe2zTfeH058 Кб, 574x604
#279 #414617
Я осознал на сколько велик Аллах, и хочу развальцевать анусы всем неверным. Какие доказательства того, что чёрных дыр не существует, су шествуют (доказательства)?
#280 #414619
>>414617
Какие есть доказательства того, что я не ебал твою мамку?
#281 #414622
>>414590

>Сможет ли человек есть пищу из инопланетных зверей и растений, имеющих ДНК из других нуклеотидов?


Да. Разложить без доступа кислорода на простые вещества, а из них - синтезировать себе пищу.

>Если все эти расщеплённые белки, нуклеиновые кислоты, полисахариды и т.д.


>будут иметь совершенно другое химическое строение, то сможет ли человеческий организм их усвоить?


Да, но гидролизом, на более простые пептиды и моносахариды.
Для этого нужна кислота, гидролиз как правило кислотный, и кислота - серная, как водоотнимающее средство.

>Если на этой планете будет такая же атмосфера, как на Земле, но


>вирусы будут иметь другую РНК/ДНК, то насколько будет опасно человеку гулять


>по планете без скафандра?


Ой бля, а вот это уже опасно. Это не просто бактериологическая угроза, но какая-то вирионная.
Даже если ДНК кремниевая, в силу того, что она достаточна мала для обнаружения
и уж тем более для нейтрализации - для защиты от подобного - только скафандр.
Если его ещё не разъест. Ну прикинь, ты такой, йоу - и врываешься в скафандре на планету
высокоразвитых но примитивных молекулярных нанороботов, разлагающих скафандры анона в один момент.
И всё то - потому что программируемая материя и молекулярные компьютеры.

> Если инопланетный вирус попадёт в клетку, то как он сможет встроиться в ДНК человека,


>ведь он имеет совершенно иное строение, чтобы потом на его основе с помощью факторов транскрипции создавалась мРНК?


Он может быть просто повредить клетку, а может заменить её ДНК своей, если например ДНК кремниевая.
Представь себе, ты попадаешь на другую планету и через время, оставаясь собой твоя ДНК постепенно превращается в кремниевую.
Почему кремниевая? Да потому что кремний четырехвалентен, как и углерод, и являясь альтернативой ему,
может образовывать кремнийорганические соединения.
Алсо, чужеродная ДНК на основе углерода, может быть просто ращеплена на нуклеотиды и усвоена клетками как пища.
В противном случае, клетки будут необратимо повреждены, и даже если остануться живучими - станут раковыми
и могут быть просто отторгнуты организмом. Такие дела.
#281 #414622
>>414590

>Сможет ли человек есть пищу из инопланетных зверей и растений, имеющих ДНК из других нуклеотидов?


Да. Разложить без доступа кислорода на простые вещества, а из них - синтезировать себе пищу.

>Если все эти расщеплённые белки, нуклеиновые кислоты, полисахариды и т.д.


>будут иметь совершенно другое химическое строение, то сможет ли человеческий организм их усвоить?


Да, но гидролизом, на более простые пептиды и моносахариды.
Для этого нужна кислота, гидролиз как правило кислотный, и кислота - серная, как водоотнимающее средство.

>Если на этой планете будет такая же атмосфера, как на Земле, но


>вирусы будут иметь другую РНК/ДНК, то насколько будет опасно человеку гулять


>по планете без скафандра?


Ой бля, а вот это уже опасно. Это не просто бактериологическая угроза, но какая-то вирионная.
Даже если ДНК кремниевая, в силу того, что она достаточна мала для обнаружения
и уж тем более для нейтрализации - для защиты от подобного - только скафандр.
Если его ещё не разъест. Ну прикинь, ты такой, йоу - и врываешься в скафандре на планету
высокоразвитых но примитивных молекулярных нанороботов, разлагающих скафандры анона в один момент.
И всё то - потому что программируемая материя и молекулярные компьютеры.

> Если инопланетный вирус попадёт в клетку, то как он сможет встроиться в ДНК человека,


>ведь он имеет совершенно иное строение, чтобы потом на его основе с помощью факторов транскрипции создавалась мРНК?


Он может быть просто повредить клетку, а может заменить её ДНК своей, если например ДНК кремниевая.
Представь себе, ты попадаешь на другую планету и через время, оставаясь собой твоя ДНК постепенно превращается в кремниевую.
Почему кремниевая? Да потому что кремний четырехвалентен, как и углерод, и являясь альтернативой ему,
может образовывать кремнийорганические соединения.
Алсо, чужеродная ДНК на основе углерода, может быть просто ращеплена на нуклеотиды и усвоена клетками как пища.
В противном случае, клетки будут необратимо повреждены, и даже если остануться живучими - станут раковыми
и могут быть просто отторгнуты организмом. Такие дела.
#282 #414627
>>414617
мультиверс официально подтвержен КОраном, ты либо жирный даун, либо даже Коран не читал
#283 #414632
Почему существуют геи? Они же не могут распостранять свои гены. Это какая-то мутация в ДНК или что?
191 Кб, 500x500
#284 #414642
>>414632
это выбор
#285 #414656
>>414644
Что такое генетическая недоработка? Почему она не передается генетически?
#286 #414661
>>414656
передается с вероятностью.
#287 #414664
>>414662

> Мы НИМОЖИМ вырабатывать чёртов витамин С


как были макаками - фруктов хватало
такой себе проёб
#288 #414665
>>414662

> Половая система засунута в моче выделительную


это ты ещё клоаку не видывал
#289 #414666
>>414662

> В отдельных клетках иногда происходят мутации и иммунитет иногда не справляется с убийством мутировавших клеток и они разрастаются в опухали


ты лучше скажи у кого с такой же продолжительностью жизни такой проблемы нет
#290 #414668
>>414667
Просто отбор на эту штуку не работает. Сам подумай, репродуктивный возраст это довольно близко к началу. В итоге ты передаешь только инфу о выживаемости до репродуктивного возраста. Если у твоих предков поголовно была предрасположенность к старческим болезням, к раку, и к чему-нибудь еще. Это не мешало им в 16-25 делать детей, что естественным отбором считалось за одобрение данного типа эволюции генома.
#291 #414670
>>414669

> любой вид


> во внешней среде


че за среда?
#292 #414672
>>414671
не, там пищи нихуя нет и условия довольно экстремальные для них
#293 #414676
>>414673

> древясиной


впринципе в тропиках есть частично съедобная древесина(после обработки каменным топором, но в большинстве случаев - онли листья, иногда кора)
>>414674

> Жуков там сожрать


отличная тема

> земля сама это тоже биомасса


нет, лучше говница наверни, там иногда недопереваренная пища встречается
#294 #414677
>>414673

>с пола


Нежелательно. Здесь речь не о микробусах, а о полноценных физических частицах, в которых эти микробусы и есть.
Упал кусок каши, к нему прилип кусок уличной грази с твоей пятки, который ты можешь увидеть воочию. Будет глупо жрать этот кусок земли.

>ветка древесина


Смотя что и как. Некоторые деревья ядовины, плюс личинки там внутри не очень круто.

>воду


Смотря что где и как. Ты слишком высокого мнения о себе и слишком низкого о своих предках.
Человек кипятил воду издревна, это раз. Во-вторых, горные ручьи и ключи это самый чистйший тип воды, который можно и нужно даже сейчас пить. Плюс экология была чище в разы-об этом важно помнить.

>годовые


В пустынях ночью ОЧЕНЬ холодно, так как песок быстрее остывает+ветрено.
Так что нет. Умернные-тропики, всегда нужно обустраивать комфорт.

>>414674
У каждого человека есть какая-либо непереносимость к стандартной еде, что уж тут говорить о нестандартной. Твое тело состоит из определенного набора соеденений, отклонения не нужны.
#295 #414691
>>414662
Ну так человек - продукт эволюции и он находится в постоянном процессе модификации, поэтому неудивительно, что таких косяков полно. Через n-ое количество поколений какие-то из этих косяков исчезнут и появятся другие. Не понимаю в чём бугурт.
#296 #414692
>>414691
мы ебанем себя раньше
#297 #414693
>>414692
Главное ИИ запилить, а так люди и не нужны.
#298 #414694
>>414691

> n-ое количество поколений


естественный отбор закончился считай, алло
#299 #414695
>>414694
В каком месте он закончился, лолка?
#300 #414698
>>414693
ИИ невозможен, маня
#301 #414699
>>414698
Ты так скозал?
#302 #414700
>>414699
Все так говорят.
#303 #414701
>>414700
Кто все, маня? Чё-то учёные со всего мира сидят и пытаются запилить ИИ, а тут у тебя все скозали, что ИИ невозможен. Ну так может расскажешь этим дуракам, что ИИ ВСЁ? А то ж они не знают!
#304 #414703
>>414701

>ученые


Малолетний дебил-научпопер с ютуба, плез.

Держи, жертва мидиа, и не позорься больше https://arxiv.org/pdf/1606.00652.pdf
#305 #414704
>>414701

>ученые со всего мира


но твои однокашники не являются учеными...
#306 #414707
>>414703
>>414704
Да что ты мне свою хуйню кидаешь, дебич? То что сегодня никто не знает как запилить ИИ не означает, что его не запилят через 100-1000 лет. Нет, он будет с пеной у рта доказывать, что ИИ невозможен, раз какой-то ноунейм так написал!
b60875 Кб, 950x608
#307 #414708
#308 #414712
>>413326 (OP)
Существует ли сон без сновидений (в фазе самого глубокого сна например)? Если да, то как это доказали?
#309 #414714
>>414698
Если запилить нейрокопию твоего мозга,(это просто сложно сделать, уровень технологий нужен покруче). То вполне себе ИИ.
#310 #414716
>>414447

>йод и прочие минералы неорганические и им насрать


Они останутся в воде, а воду сливают.
#311 #414717
Я знаю, что здесь сидит достаточно большое количество умных и образованных людей. И здесь наверняка найдутся люди, которые мне нужны. Дело в том, что я пишу сюжет для научной фантастики, но сам, имея богатое воображение, имею представление об окружающем мире достаточно посредственное, разбираясь в химии, физике и биологии только на уровне представлений, которые мне дают соответствующие научно-популярные фильмы, статьи и прочее (чем я отнюдь не горжусь). Мне нужны очень образованные и интересующиеся этими сферами люди, прежде всего, научные оптимисты и реалисты, которые не кричат о мёртвом космосе и невозможности существования жизни при 500 градусах по цельсию (по крайней мере, безосновательно). Мне нужны люди с исследовательской позицией, которые могут сослаться на конкретные работы. Да что уж там, даже люди, просто увлекающиеся научной фантастикой, будут желанны! От вас почти ничего не потребуется: в течение нескольких лет я буду очень редко задавать вам глупые вопросы о возможности или невозможности определённого явления, а вы будете в желаемой вами форме давать мне ответы на эти вопросы: кто-то просто может сказать "да, такое явление возможно" и я просто поверю вам как авторитетному человеку, кто-то может сказать "нет, невозможно" и объяснить почему это так в форме "для полного дурака и профана", а кто-то при желании ещё и подкинет мне пару исследований. Работа будет проходить очень медленно и главное, что от вас требуется — просто оставаться на связи, в остальном от вас не требуется никаких усилий и затрат времени. Неплохо было бы собраться в телеграмме, но если нет желания его ставить, то ок.
#312 #414718
>>414695

> В каком месте


например в кувезе
#313 #414719
>>414717
спрашивай уже вопрос
конфочку тоже можешь кинуть, не забанят, лул только насрут в неё знатно
#314 #414720
>>414719
Я же говорю, что, во-первых, мне нужно на постоянной основе в течение нескольких лет задавать эти вопросы (которые у меня сейчас только в общих чертах в голове), во-вторых, чтобы обойтись без типичных двощерских срачей, ненависти и метаний говн, если это будет организовано в форме конфы, просто нормальный диалог с фактами и литературой (хотя никто, опять же, не принуждает к нему, можно ограничиться ответом "да" или "нет", максимально лаконично).
#315 #414721
>>414720
В плане по мере их возникновения.
#316 #414723
>>414720
тебе в аналогичный тред в /spc/
#317 #414724
>>414723
Но вопросы, в основном, не про космос. Эх, ладно, юзлесс. Всё равно я нашёл этот тред, буду задавать здесь, разницы никакой.
#318 #414727
>>414619

>Какие есть доказательства того, что я не ебал твою мамку?


Папаша, хватит ходить по комнате в рваных труханах и пить травы какие-то. Ответь лучше на вопрос? Какие есть альтернативы чёрным дыреням?
>>414627

>мультиверс официально подтвержен КОраном, ты либо жирный даун, либо даже Коран не чита


Аллах настолько велик, что сам написал Священный Коран. Не тебе решать вместо Аллаха что в Коране написанно.
#319 #414728
>>414695
В том, что инвалиды не умирают, а общество им помогает.
#320 #414729
>>414717
Могу дать телеграмм, но у самого не намного больше знаний, чем у тебя. Да и зачем тебе это в научной фантастике? Если ты пишешь что то типа марсианина, то да, тебе нужны хоть какие-то знания. Но для аналога звездных войн главное сюжет и атмосфера, а не возможность существования светового меча ИРЛ.
#321 #414730
>>414677

>Люди кипятили воду издревле


А как они додумались до этого? Раньше же никто не знал про микробы и болезни. Я думал, ее начали кипятить только после того, как узнали что такое микробы
#322 #414731
>>414712
бамп
#323 #414735
Вот есть у нас кусок металла, допустим. Мы его нагреваем до определенной температуры и он начинает светиться. Значит, испускает фотоны и/или волны. Откуда они берутся?
Вроде как, вроде все тела светятся в инфракрасном, неужели нагревание как-то влияет на длину волны? Если да, то как это происходит?
#324 #414736
>>414735
У атомов есть энергетические уровни, если собрать очень много атомов, то и у большого тела, должны быть эти уровни, только их будет очень очень много. И тело будет переходить между ними, пытаясь остыть. Будет испускать какой-то свет. Впервые точно описать спектр тела, смог Планк.https://ru.wikipedia.org/wiki/Формула_Планка
#325 #414738
>>414730

> Раньше же никто не знал про микробы и болезни.


> и болезни


ой всё
#326 #414739
>>414730
Эмпирически
#327 #414740
>>414730
От нехуй делать.
#328 #414781
От чего зависит температура плавления веществ при одинаковом давлении?
#329 #414782
>>413326 (OP)
Хуй знает где спрашиавть, но, в общем, надо диплом добить.
В одном из разделов надо посчитать отказоустойчивость двух сетей:
- два сервера (один зеркалирующий)
- ебанный кластер с репликацией и автомасштабированием.

Как бы в подошли к такой задаче? Просто я только всякие кластеры собирать умею, а обосновать хуй знает как.

Выручай, анон.
#330 #414786
>>414781
Энтропия плавления, и энтальпия плавления
#331 #414818
Что будет если посадить в прилегающие клетки хомячков и гадюку? Разделить их решеткой, чтобы чувствовали запах друг друга. Более того, сделать защищенные каналы, чтобы они могли забегать в другую клетку, но без прямого контакта. И наконец сыпать корм хомячкам в такое место, чтобы они должны были пробежать по каналу, проходящему через территорию гадюки. Рядом можно еще посадить умного попугая, чтобы он смотрел и ловил лулзы.
Sci17 Кб, 193x103
#332 #414826
Почему пикрилейтед не будет работать?. С самого детства волнует вопрос
#333 #414828
>>414826
Палка мешает.
#334 #414829
>>414826
Если поставить электромагнит, который постоянно включать и отключать.
Если плечо у крана будет пружинным.
То за счёт силы тяжести можно будет управлять центром равнодействия сил и при не нулевой силе трения с землёй, оно начнёт перемежаться.
#335 #414831
>>414829
А вот без всего это. Если магнит будет отдельно он будет притягивать машину и она будет двигаться. А если магнит и машина будут одним целым, то уже не двигается. Что за магия?
tmpE73-1936 Кб, 342x432
#336 #414835
#337 #414836
>>414782
Бамп
#338 #414837
>>414829
Можно было бы использовать магнитную заслонку из какого-нибудь экранирующего магнитное поле материала.
Но эта заслонка должна была бы растопыривать силовые линии магнитного поля куда-то к магниту.
Интересно вот только, чтобы вдавить эту заслонку в условиях притяжения магнитом - нужна ли энергия?
#339 #414838
>>414736
У тебя компот в голове.
#340 #414841
Почему когда я смотрю на обыкновенное оконное стекло в нем не наблюдается явление интерференции, как при наблюдении за мыльным пузырем? И что сделать, чтобы появилась интерференционная картина в оконном стекле?
#341 #414842
>>414841
Потому что стекло толстое. Чтобы наблюдать такую картину, нужно чтобы толщина была сопоставима с длиной волны видимого света. Плюс стекло не такое ровное.
#342 #414843
>>414841

>что сделать, чтобы появилась интерференционная картина в оконном стекле?


Сделать его дохуя тоньше.
#343 #414849
що будить если в центре урагана взорвать ядерную ракету????
#344 #414850
>>414849

> що будить


взрыв атомной бомбы в урагане
#345 #414851
>>414850
ау рогсн не рассасется?
#346 #414861
>>414849
Моделировать надо. Возьми компьютер и смоделируй.
#347 #414862
Анон. в тысячный раз задам вопрос по поводу эксперимента юнга. везде две крайности. либо ёбаный матан на английском, либо обьяснение для дошкольников.
интересуют следующие вопросы

1. электроны пускали по одному. каким устройством и с какой частотой?
2. какого устройство наблюдателя.

и самый интересный
3. как был реализован принцип отложенного выбора. интересует именно факт получения и уничтожения данных с наблюдателя. факт которого влиял на результат.
#348 #414863
>>414035
спасибо тебе за вопрос, сам не понимал. это реально не очевидно
#349 #414864
>>413999

>русин из эндогамного рода.


конечно может быть здоров, но судя по дереву. такое веселье не три покаления назад началось. и про шансы говорить сложно
#350 #414865
>>414862
Для этого надо статью расшарить оригинальную. Можешь посмотреть на что ссылаются на вики. И потом по аторам или DOI поискать саму статью. У Фейнмана в книге описано, мол смотрели как рассеивается фотон на электроне.
#351 #414866
>>414865

> рассеивается фотон на электроне


?

всмысле следят за отклонением фотона от электрона?
#352 #414867
>>414866
Это значит светят на дырку, и если что-то отразилось, значит это отразил электрон. Который мимопролетал
#353 #414868
>>414862
Берёшь опыт Юнга, и повторяешь его. Тебе понадобятся две щели.
А для разгона электрона тебе надо внешнее электрическое поле
и работа, большая чем работа выхода электрона из материала.
#354 #414872
Тут карочи читаю одного шизика.

Как известно, при любых процессах в изолированной системе энтропия всегда растет. Если температура воды будет равна 0, а лед — достаточно охлажденным для того, чтобы отнять у воды тепло фазового перехода, то одновременно с ростом энтропии эта хаотическая смесь обратится в упорядоченный кристалл. Таким образом, оказывается, что порядок из хаоса может возникнуть в изолированной системе одновременно с ростом энтропии.

Всё правильно?
#355 #414876
>>414872
лел. И почему он взял изолированную систему. Там есть проблема с давлением. Но вообще вопрос ты странный задал. Знал бы термодинамику хоть немного, понимал бы что к чему. Энтропия действительно вырастет. Смотри сам, у тебя энтропия будет при кристаллизации уменьшаться, но тепло которое будет греть кусок льда, будет увеличивать его в нём. Если простенько посчитать, то выйдет что в сумме увеличится.
#356 #414881
>>414782
Бамп.
#357 #414907
Как получить водяной лед с помощью сухого льда?
#358 #414909
>>414907
Главное не смешивать, а то CO2 растворится в воде, и будет не круто. Берешь бутылку воды, закрываешь её. И засыпаешь её сухим льдом. Через пару часиков у тебя бутылка льда.
#359 #414910
Вопрос. Вот есть в математике доказательство несоизмеримости, теорема Абеля-Руффини, теоремы Гёделя. Касаются они совершенно различных математических объектов, но объединяет их то, что все три, по сути, являются "доказательствами невозможности" - и после их появления, эта самая математика делала резкий уклон в сторону. Иначе, эти доказательства порождали радикально новые пути развития математической мысли.

Есть ли в математике ещё примеры подобных "доказательств невозможности", по фундаментальности и исторической значимости сравнимых с уже упомянутыми?
#360 #414913
Сап всем обитателям данного треда. Интересует тема о перемещении во времени. Недавно, вроде американский, учёный изложил свою теорию. В сети можно было найти. Короче говоря, заинтересовало. Посоветуйте что можно почитать, да и вообще с чего начать.
#361 #414917
>>414910
Как алгебраист говорю тебе, что ты написал какой-то треш. Обколются своим научпопом а потом ябут друг друга в гротендика.
andromedagendler[1]127 Кб, 800x600
#362 #414920
Внимание вопрос!
Господа Знатоки.
Как вкатиться в Астрофизику? (По дешману)(Или всякие там Астрономии) (Спектральные анализы всякие)
Смотреть на скопления с умным видом, помечать что-то в тетрадочке. Мечтать о колонизации Марса.
На своем 26.9 лвле, перепробовал множество занятий, сейчас вот новостей об освоении космоса обчитался.
Приветствуется короткий (подробный) ответ куда идти и что курить.
#363 #414921
Может ли наша вселенная развлетлиться на параллейные вселенные? Если может, то как это происходит?
#364 #414923
>>414921
Может, и даже скорее всего это и делает. Гугли многомировую интерпретацию квантовой механики.
#365 #414924
>>414920
Я боюсь это сложно сделать с полным погружением. Это надо было класса с седьмого упарывать физику, и про математику не забывать.
Но хрен знает насколько легко тебе будет всё даваться.
#366 #414925
>>414923

>Согласно гипотезе создателя многомировой интерпретации квантовой механики Хью Эверетта, мы живем во Вселенной, точнее в мультивселенной, в которой постоянно рождается и ответвляется множество последовательных миров, в каждом из которых присутствует другая версия вас.


А теперь вопрос: каким образом? То-есть просходит одновременно 2 события, которые противоречат друг другу и по ним создаются новые мультивселенные? Типа как кот Шреддингера одновременно и жив и мертв и создаются вселенные, где он выживает и где он умирает.
#367 #414940
>>414925
Они не противоречат, так как друг на друга не влияют. Когда мы говорим про состояние суперпозиции для фотона, всем норм, для электрона, тоже всем норм, для фуллерена, всем норм, для ансамбля частиц, всем норм, для большого кристалла с количеством в миллиарды атомов, всем тоже норм. И дальше очень простое заключение возникает, что если ты взаимодействуешь с чем-то что находится в суперпозиции, то ты тоже должен стать в суперпозиции, это наглядно доказывается с помощью уравнения Шредингера, и его линейных свойств. Но вот тут часть научных работников спокойно принимает это, и говорит мол ну да, Эверетт прав, а часть начинает срать кирпичами и не хотят признавать такое, придумывают лютые интерпретации, которые у них нарушают либо детерменизм, либо локальность.
#368 #414943
>>414917
Алгебраист, а ты вообще в курсе, последовательность каких именно публикаций, когда и кем (это был не один человек, разумеется) написанных, привела к самому возникновению теории групп, как таковой, и к последующему переопределению самого понятия "алгебра", как такового? Ты вообще в первоисточниках хоть когда-нибудь в своей жизни ковырялся, алгебраист?
#369 #414944
>>414862
Об этом никто не говорит, потому что этот эксперимент придумали евреи.
#370 #414946
Как фотон может иметь импульс давление света, если у него нет массы? Они свои работы под мухоморами пишут чтоли?
#371 #414947
>>414944
шепотом смотрите, щас смешно будет.
Эй, рус, из чего атомы состоят? Поделись знаниями.
#372 #414948
Сап sci. Предположим, в направление солнечной системы летит черная дырень массой 10000 солнц, с гигантской скоростью 1350 км/с, примерно попендикулярно плоскости, в которой крутятся планеты. Вопрос - сможет ли наблюдатель, у которого есть только херовый телескоп, из которого даже лун юпитера не разглядишь, увидеть её, или же просто заподозрить неладное на основе поведения других объектов солнечной системы до того, как эта дырень выбьет землю с орбиты солнца?
#373 #414951
>>414946
Если коротко - у фотона нет массы покоя, но масса есть. Потому как он имеет энергию, а так как E=mc^2, то m=E/c^2. Под массой покоя подразумевается масса остановленного фотона. Фотон то конечно остановить можно, но Eфотон=hv, где h - чья-то постоянная, а v - частота, v=1/T (T - период), a T=λ/Vфотона, (λ - длинна волны, Vфотона - скорость фотона). Так как пари делении на ноль получается бесконечность, а при делении на бесконечность ноль, то частота остановленного фотона равна нулю, а значит и энергия равна нулю. Масса получается тоже.
#374 #414952
>>414946
Потому что импульс это не mV^2/2. Тебя наебали.
#375 #414956
>>414951

>нет массы покоя, но масса есть.


Статью Окуня в УФНе скинуть? Или сам найдешь?
image2 Кб, 189x42
#376 #414958
>>414952
Подставь вместо массы 0. Заметь что импульс остался.
#377 #414962
>>414958
Разве у фотона не переменная масса в каждом полупериоде электромагнитного колебания?
#378 #414964
>>414962
Я честно не знаю как там КЭДе, но думаю что нет,
Снимок экрана в 2017-06-12 21-58-3034 Кб, 362x356
#379 #414966
>>413326 (OP)
Какие есть современные утопические идеи, основанные на клонировании людей? А то кроме отсылок к 1984 и "о дивный новый мир" ничего нового найти не могу, хотя за это время технологии давно шагнули вперёд. Есть какой-то около политический/утопический трактар, который использует клонирование человеков?
#380 #414967
Ничто существует?
И если ничто существует, его бесконечно много?
#381 #414968
>>414967
Это вопрос скорее философский.
#382 #414970
>>414348
Кстати, если вчитаться в статью, то там не все электроны 52 электрона отлетели при ионизации, а всего 47 электронов отлетело, при этом, возвращаясь назад, атом схлопнулся в чёрную дыру от эффекта Оже, возникающего при этом.
#383 #414972
>>414966
Ну, однояйцевые близнецы тебя устроят?
#384 #414974
>>414972

>однояйцевые близнецы


Это название романа/фильма или ты про сам феномен? Если второе, то это неинтересно. Интересует именно человеческое клонирование, когда берут соматические ядра и в денуклеированные яйцеклетки подсаживают. Мой препод по молекулярной биологии говорил, что это возможно сделать в том числе и с людьми, вот и хотелось бы почитать, если ли какие-то идеи по поводу того, как с помощью этой технологии огранизовать идеальное общество, какие уже варианты предложены, кроме мира Хаксли (у него немного устаревшие технологии).
#385 #414979
Когда в рашке сделают квантовый компьютер?
#386 #414982
>>414979
Давно уже есть.
#387 #414983
>>414982
нету ничего
#388 #414984
Время - это пространство?
#389 #414986
>>414984
Вопрос в том, что ты вообще считаешь пространством. Ибо узнаешь ты, что что-то ближе к тебе, только потому что оно иначе взаимодействует с тобой, когда далеко. Пространство, это субъективное восприятие трех переменных, которые возникают в законах физики. Ты вполне мог бы воспринимать расстояние до предмета его цветом, а цвет видить как расстояние до предмета, но очевидно это было бы неудобно для восприятия. Время входит в законы физики на тех же правах что и другие три пространственные переменные(почти, есть свои оговорки со знаками, и в квантах немного заморочек, но тоже можно впихать). Но ощущаешь время ты иначе, нежели пространственные величины.

Если не хочешь учить матан и физику, для того чтобы это понять, попробуй ЛСД, оно поменяет на время твоё восприятие, и после этого ты не будешь уже настолько привязан к абсолютности твоего восприятиясвязанного с тем,
что ничего другого ты никогда и не видел,
эдакий карго-культ твоего восприятия реальности
#390 #414987
>>414974

>Это название романа/фильма или ты про сам феномен?


Это не феномен - это явление.

>Если второе, то это неинтересно.


Ну, по мере дифференциации и расхождений в развитии их и различия видны. Они формируются таким образом эти различия.

>Интересует именно человеческое клонирование, когда берут соматические ядра и в денуклеированные яйцеклетки подсаживают.


В соматических клетках теломеры укороченные и лимит Хейфлика ещё сюда приложу.

Поэтому, для полноценного клонирования - надо бы проводить их иммортализацию >>408961 >>409159
либо же юзать ядра стволовых клеток, направляя их при помощи различных вещество по определённому пути дифференциации.
Стволовые клетки имеются в костном мозге. Их называют гемоцитобластами.
Однако гемоцитобласты являются плюрипотентными стволовыми клетками.
Плюрипотентные клетки могут дифференцироваться во все типы клеток,
кроме клеток внезародышевых органов (плаценты и желточного мешка).
А вот оплодотворённые яйцеклетки животных являются тотипотентными.

А можно было бы просто секвенаторами >>408432 секвенировать ДНК множества соматических клеток,
оцифровать её и при помощи молекулярных компьютеров, сложить из обработанной ДНК - ДНК стволовых клеток,
а потом записать их, например в кремнийорганических соединениях, и использовать как эталонную матрицу
для репликации ДНК при воссоздании полнофункциональных ядер стволовых клеток определённого генотипа.

>Мой препод по молекулярной биологии говорил, что это возможно сделать в том числе и с людьми


Ну вот нахрена тебе клон? Разве что как ёмкость для органов что-ли? Можно и свиней юзать для их выращивания:
https://lenta.ru/news/2017/04/17/cancer/

>вот и хотелось бы почитать, если ли какие-то идеи по поводу того, как с помощью этой технологии огранизовать идеальное общество


Каким ты видишь идеальное общество? Без болячек что-ли? Так это результаты развития организмов в среде обитания.
Да, вполне возможно это обусловленно тенденциями развития, которые в свою очередь могут быть прошиты на генном уровне.
Но не думаешь ли ты, что манипуляции с генами могут позволить создать идеальный геном,
который в состоянии был бы исключить различные перекосы в развитии, например рак,
и обладал бы максимальной устойчивостью, экстропией и даже неуязвимостью?
Это всё по поводу индивидуального развития организма. А что касается общества - тут уже социологии наверни.
#390 #414987
>>414974

>Это название романа/фильма или ты про сам феномен?


Это не феномен - это явление.

>Если второе, то это неинтересно.


Ну, по мере дифференциации и расхождений в развитии их и различия видны. Они формируются таким образом эти различия.

>Интересует именно человеческое клонирование, когда берут соматические ядра и в денуклеированные яйцеклетки подсаживают.


В соматических клетках теломеры укороченные и лимит Хейфлика ещё сюда приложу.

Поэтому, для полноценного клонирования - надо бы проводить их иммортализацию >>408961 >>409159
либо же юзать ядра стволовых клеток, направляя их при помощи различных вещество по определённому пути дифференциации.
Стволовые клетки имеются в костном мозге. Их называют гемоцитобластами.
Однако гемоцитобласты являются плюрипотентными стволовыми клетками.
Плюрипотентные клетки могут дифференцироваться во все типы клеток,
кроме клеток внезародышевых органов (плаценты и желточного мешка).
А вот оплодотворённые яйцеклетки животных являются тотипотентными.

А можно было бы просто секвенаторами >>408432 секвенировать ДНК множества соматических клеток,
оцифровать её и при помощи молекулярных компьютеров, сложить из обработанной ДНК - ДНК стволовых клеток,
а потом записать их, например в кремнийорганических соединениях, и использовать как эталонную матрицу
для репликации ДНК при воссоздании полнофункциональных ядер стволовых клеток определённого генотипа.

>Мой препод по молекулярной биологии говорил, что это возможно сделать в том числе и с людьми


Ну вот нахрена тебе клон? Разве что как ёмкость для органов что-ли? Можно и свиней юзать для их выращивания:
https://lenta.ru/news/2017/04/17/cancer/

>вот и хотелось бы почитать, если ли какие-то идеи по поводу того, как с помощью этой технологии огранизовать идеальное общество


Каким ты видишь идеальное общество? Без болячек что-ли? Так это результаты развития организмов в среде обитания.
Да, вполне возможно это обусловленно тенденциями развития, которые в свою очередь могут быть прошиты на генном уровне.
Но не думаешь ли ты, что манипуляции с генами могут позволить создать идеальный геном,
который в состоянии был бы исключить различные перекосы в развитии, например рак,
и обладал бы максимальной устойчивостью, экстропией и даже неуязвимостью?
Это всё по поводу индивидуального развития организма. А что касается общества - тут уже социологии наверни.
#391 #414989
>>414984
Временной компонентой вектора четырёхимпульса является (с точностью до множителя) полная энергия материальной точки.
#392 #414990
>>414987

>либо же юзать ядра стволовых клеток, направляя их при помощи различных вещество по определённому пути дифференциации.


This. Препод по молекулярке об этом и говорил. Берёшь человеческую тушку, выдираешь у неё из кожи стволовых клеток, денуклеируешь, вставляешь ядро в яйцеклетку, штампуешь клонов в промышленных инкубаторах, у одного из таких клонов вырываешь из кожи стволовых клеток, цикл замкнулся.

> Можно и свиней юзать для их выращивания


Мне нужны люди. Я же тут идеальное общество строю, а не очередной киберпанк с корпорациями и продажей органов на чёрном рынке.

>Каким ты видишь идеальное общество?


Я как раз и хотел узнать, какие существуют на этот счёт концепции.

>манипуляции с генами могут позволить создать идеальный геном


Это не особенно интересно. Волнует именно то, каким видят общество с применением таких технологий, причём важно как можно более исчерпывающее описание (то есть чтобы я охватил все из предложенных концепций).
Просто я придумал одну концепцию и мне хочется понять, насколько я сплагиатился. Ведь не мог же я что-то оригинальное создать, наверняка кто-нибудь уже писал об этом и сделал это лучше. Вот и хотелось бы узнать, кто. А для этого нужно исчерпывающее описание разных вариантов утопических сообществ, базирующихся на клонировании человека.
#393 #414992
>>414990

>Берёшь человеческую тушку, выдираешь у неё из кожи стволовых клеток,


кстати в жировой ткани тоже стволовые клетки обитают.
http://biopro-st.com/ru/patsientam/stvolovye-kletki-zhirovoj-tkani-okazalis-naibolee-stabilnymi

>денуклеируешь, вставляешь ядро в яйцеклетку,


>штампуешь клонов в промышленных инкубаторах,


А вот тут уже не... Потому что плюрипотентность.
Для беременности в инкубаторе тебе прийдётся воссоздавать условия роста плаценты
и желточного мешка, если ещё и не пуповины, лол.

>у одного из таких клонов вырываешь из кожи стволовых клеток, цикл замкнулся.


В коже и жировой ткани - теломеры поменьше будут...
Алсо, проще реализовать размножение клонов - почкованием, и пускай они будут пупками перевязаны
и как-бы отростать от одной здоровой такой плаценты-матки, лол.

>Я как раз и хотел узнать, какие существуют на этот счёт концепции.


Ну... Идёшь в гугл, и узнаёшь... А если хочешь правду, то...
#394 #414993
>>414992

>Алсо, проще реализовать


Конкретная реализация в общем-то не имеет значения.

>идёшь в гугол


Я как раз оттуда и пришёл. Особо никакой информации не нашёл, в википедии (английской) тоже не густо.
telephant #395 #415013
Подскажите, можно ли без учетки смотреть статьи в Scopus, или есть какие-нибудь обходные пути?
#396 #415014
>>414948
-Диванный кто-то там вошел в тред.
-Диванный кто-то там пишет: крч, поидее мы наверное можем не заметить настолько близкого искривления пространства. Все что далеко из говноскопа так-же не заметить.
#397 #415119
Каким образом можно рассчитать мощность двигателя, который удерживает на каком-либо расстоянии от земли тело массой m при отсутствии опоры?
#398 #415160
>>415119
Если нет опоры, то единственным способом удержать что либо яляется реактивное движение. Кроме того надо учитывать, что на двигатель не также действует как и на тело гравитация. Следовательно двигатель должен удерживать массу тело и собственную массу. Поэтому для решения необходима знать массу двигателя.
#399 #415161
Помогите понять Теорему Байеса.
Допустим, у нас есть коробка в ней 7 шаров, 3 белых и 4 черных. Мы ищем вероятность для белого шара.
Найти совместную вероятность просто, нужно просто достать шар, если он будет белый, то (3/7) умножаем на (3/6)
А как находится полная вероятность?
Необходимо достать еще один шар и на основе этого наблюдения уже производить вычисление?
#400 #415163
>>415160
Масса m включает массу двигателя. И вообще, двигатель не обязательно реактивный, это вполне может быть пропеллер или сверхпроводники.
#401 #415168
>>415119
Просто, она равна 0. Т.к. не совершается работа.
#402 #415169
Насколько может быть справедлива гипотеза, согласно которой в момент смерти субъективное восприятие времени вытягивается в бесконечность?
thepowerofrandomness74 Кб, 460x259
#403 #415172
Бля, а че ученые эти не додумались вот до чего. Берем, и начинаем рандомизировать байты. Байты переводим в чертежи. И так пока не срандомизируется FTL-движок. Потом делаем его и пиздуем к Андромеде, отвечать на самый главный вопрос человечества: Зачем мы тут? Одиноки ли мы?.

>Где мощность будешь брать?


Используем доступные суперкомпы при университетах + добровольные вычисления волонтеров + можно у Трампа попросить бабла на компы.

>А как отличишь говно от концепций?


Нанимаем индусов которые будут следить за каждым срандомизированным чертежем + возможности машинного обучения, чтобы распознавали что-то похожее на двигатель хуйню. Все похожее отправляем ученым, те изучают и пробуют концепцию в деле.

>индусов не хватит


Поэтому эту хуйню надо сделать популярной, чтобы все человечество объединилось и рандомизировало байты у себя на компе + распознавало концепции.
#404 #415180
>>415172
Чем-то похоже, не пому чем, но похоже - на мирогенерации тред.
Я где-то видел статью, не могу найти, короче там описывался простой алгоритм,
обработки нагенерированной рандомной хуиты, но не простой обработки,
а со снижением энтропии рандома всего того, что нагенерировано случайным образом.
Причём так, или иначе всё сводится к одному и тому же результату,
а по мере углубления - растёт сложность и скорость.
Алгоритм примерно, такой.
Генерятся два числа, чётное и нечётное, а потом перемножаются, или суммируются там, короче обрабатываются.
И типа, если одно четное, другое нечетное - результат четный. Если два нечетных - Нечетное. Если два чётных - Четное.
Дальше, по каскадам, вероятность чётных увеличивается. И это снижает энтропию, давая в итоге чётное.
Но если выпадает нечетное - вся структура значений копируется, и представляет из себя уникальный результат,
удовлетворяющий какому-то там условию.
Конечно же этот результат может быть оптимизирован, но прикол именно в структуре значений, и их комбинации,
которая может быть модульной, и соответствовать после некоторой корректировки определённым критериям.
И это всё может быть применено для поиска, распознавания и исследования нагенерированной хуиты.
Функции обработки переменных негенерённых значений могут быть многовходовыми,
могут предствавлять из себя системы функций или даже непрерывные функции, кодируемые этими системами.
Всё это вместе и вперемешку - как-бы порождает животворящую когнитивную синергию на принципах конструктивизма.
Можешь заюзать ещё и GRID'ы и ASICи, лол.
#405 #415182
>>415168
Прекрасно. Наверное, пропеллер у вертолета может не вертеться, если он не поднимается, а просто висит в воздухе. И топливо не тратится, если реактивный ранец просто висит на месте, так? И когда с помощью электромагнита поднимают железку, пускать ток по нему не обязательно. Так?
#406 #415183
>>415172
Количество вариантов будет огромным. Порядка 1010^10/sub]. Чтобы ты понимал, на сколько это мать твою много - количество времени планка, прошедших с, собственно, эпохи планка - =2.21013749×1062 (4×1017, если в секундах. И ты ведь сможешь загуглить, что есть эпоха планка и время планка, так?).
#407 #415184
>>415183
Я хотел написать, порядка 1010^10 вариантов, но что-то пошло не так. Добавлю, что скорее всего показатель степени будет на порядки больше.
#408 #415191
Движение тока происходит от минуса к плюсу?
#409 #415193
>>415161
Анончик, совсем не разбираешься? Тут вопросы сложнее обсуждаются обычно.
#410 #415194
>>415191
От плюса к минусу.
#411 #415196
>>415194
Нет, от минуса к плюсу.
#412 #415197
Я раньше думал что квантовое запутывание/телепортация позволит передавать инфу, но наткнулся на многие опровержения типа

>Действительно, согласно современным представлениям, передача состояний между сцепленными фотонами происходит мгновенно, таким образом, может возникнуть ощущение, что и информация передаётся мгновенно. Это, однако, не так, поскольку хотя состояние и было передано, прочитать его, расшифровав послание, можно только после передачи дополнительной информации о том, каковы же поляризации двух фотонов, находящихся у источника. Эта дополнительная информация передаётся по классическим каналам связи и скорость её передачи превышать скорость света не может.



Также прочитал это https://geektimes.ru/post/279050/
но все равно нихуя не понял. Кто-то может проще объяснить почему нельзя передавать инфу,
#413 #415199
>>415196
Ты из школы ушел после 4-го чтоли?
#414 #415200
>>415197
Я могу, но бля, это так уныло каждый раз объяснять, и так уныло когда появляются дополнительные вопросы. Не понимаю, почему люди лезут в кванты, хотя даже не поймут уравнение Шредингера. Госпади, люди механику то не знают, но начинают лезть в кванты с вопросами, чтобы ответить на которые перед этим надо объяснять еще разные понятия физики.

Совет, открой учебник физики, там не обрывисто а последовательно всё.
#415 #415201
>>415183
10^100 вариантов
1 человек за минуту 30 чертежей
1 человек за час 1800 чертежей
1 человек за день (13 часов работы) 23400 чертежей
1 человек за год 8.54110^6 чертежей
Скажем 35% населения будут этим заниматься, это 2.57145
10^9 человек
За год 35% населения может просмотреть 2.196275445 × 10^16 чертежей
Значит понадобиться 4.5 * 10^83
Цифра довоольно большая, но:
1)у тебя цифра с потолка, слишком большое кол-во
2)нужно учитывать что FTL-движок мы можем найти и на десятом, и на миллионом чертеже, не обязательно перебирать все.
3)во время рандомизирования чертежей мы найдем чертеж машины, которая может клонировать людей. После берем и клонируем индусов пачками, расселяем их по солнечной системе, и пусть себе рандомизируют байты на Марсе или на Венере.
#416 #415202
>>415200

>Я могу, но бля, это так уныло каждый раз объяснять


Напиши один раз и сохрани в текстовый файлик чтоб кописастить. Мне просто оъясняют типа вот есть левая перчатка и правая. Кладем в ящики и один отправялем в америку. Ты открыл и видишь что у тебя левая перчатка, а в америке соответственно правая. Но какая же это запутанность и нахуй она нужна если так инфу не передать. Я думал что если перчатке в америке отрезать палец то и на моей он пропадет. Если нет то так можо вместо квантов деньги отправлять. Было 100 рублей часть отправили в магадан. Смотришь - у тебя 15 р, значит отправили 85 рублей. АХУЕННАЯ ЗАПУТАННОСТЬ которая нахуй не всралась.
#417 #415203
>>415201
понадобиться 4.5 10^83 года
фикс
#418 #415204
>>415202
У меня книга есть "Квантовая теория поля в двух словах". Я думаю можешь её почитать. Как-нибудь попробую накатать текст по поводу запутанности.
#419 #415205
>>415204

>в двух словах


На 500 страниц?
image7 Кб, 314x129
#420 #415206
>>415205
На 616
#421 #415217
>>415119
Универсального способа нет при такой постановке задачи, но можно попытаться прикинуть если следовать следующим шагам:
1. Выбираешь время на которое нужно удержать груз.
2. Выбираешь окружающую среду (воздух, вакуум и тп)
3. Выбираешь тип двигателя (пропеллер, реактив и т.п.).
4. Вычисляешь массу топлива в зависимости от двигателя и времени полета и прибавляешь к общей массе.
5. Гуглишь вычисление мощности для конкретной схемы двигателя с учетом того, что со временем топливо будет уменьшаться, масса падать и мощность тоже (интегрируй Люк).
1444856694264048 Кб, 680x853
#423 #415221
>>415218
Спасибо, посмеялся с этого лженауканья.
#424 #415225
>>415218
на 6 минуте вырубил потому что он льет воду, а что такое лженаука так и не объяснил к шестой минуте. если для пояснения одного слова нужно 13 минут, то поясняющий конченый дибил
#425 #415226
>>413411
можно сказать что это второй цвет после желтого на флаге украины.
#426 #415230
Научная статья. Как и нахуя? Есть ли профиты
#427 #415231
>>415230
Что вообще значит "как и нахуя?"
#428 #415243
>>415169
Никто до сих пор не знает что такое сознание и как это работает. Так что сложно судить о бесконечности.
#429 #415255
>>413326 (OP)
Нахуя вам 2 треда вопросов? В каком сидят питухи а в каком уважаемые люди?
#430 #415256
>>415248
От сжатия вселенной же
.
8732 Кб, 128x104
#431 #415261
>>413326 (OP)
Поясните за относительность движения. Это же ведь просто методологический приём, кастыль да? В реальности некоторые движения не относительны(как минимум движения человека)? Ведь если это не так то получается какая то хуита. Относительность движения говорит, что когда А движется относительно В то В движется относительно А и бессмысленно утверждать, что одно из них находится в движении а другое в состоянии покоя.
Предположим я вижу, как человек идёт к машине, здравый смысл говорит мне, что это машина покоится, а человек движется к ней. Но физика говорит мне, что утверждение "машина движется к человеку" столь же верно. Предположим, что этот человек осознанно поставил себе цель подойти к машине и я это знаю(ну он поделился со мной своими планами допустим). Моё чисто обыденное описание увиденного будет - человек поставил себе цель подойти к автомобилю и передвижение его тела которое я наблюдаю есть реализация этой цели. Но что же получается если я вспоминаю про относительность движения. Моё утверждение о том, что движение тела этого человека является целенаправленным оказывается бессмысленным ведь утверждение, что это он находится в состоянии покоя, а машина движется к нему оказывается столь же верным. Что же это выходи то блять!? это не я сейчас стучу по клавиатуре, это кнопки клавиатуры стукаются об мои пальцы в соответствии с тем порядком который я задумал... либо всё таки это я стучу по клавиатуре разницы нет!
#432 #415262
>>415261
Это не прием. Ведь если вы будуту с машиной обмениваться сигналами, то выяснится, что в машине замедлилось время. Что забавнее, для человека в машине, время замедлится у тебя, ведь ты движешься относительно него.
#433 #415263
>>415262
Получается я своей волей двигаю вселенную?
#434 #415265
>>415263
Не, твоя воля это полная хуита, и вселенной похуй на тебя твоей воли вообще нет
#435 #415266
Меня волнует соотношение моего ощущения воли и реальности, если до кого не дошло. Я привык думать, что когда я играя в футбол и бегу к мячу то это я движусь к нему - я поставил себе цель подбежать к нему. Но выходит, что это необязательно так и ничто не мешает мне сказать, что я на самом деле покоюсь и это мяч движется ко мне(вместе со всем окружением) просто следствии того что я захотел что бы он оказался рядом с моим телом. Бред же, но вроде как прямо вытекающий из относительности движения.
#436 #415267
>>415265
Получается что я двигаю своё тело?
#437 #415269
>>415267
Тело двигает само себя, ты просто пассажир.
#438 #415270
>>415266
Ты несешь хуиту. Никакого бреда тут нет.
#439 #415271
>>415270
С каких пор принцип относительности стал бредом?
#440 #415273
>>415266
Во вселеной правят поля, а полям вообще похуй где ты. Полям важно лишь |r1-r2|, тупо расстояние. Поэтому ты просто меняешь расстояние между тобой и мячом.
Пространство-время вещь изотропная во всех направлениях, именно по этим причинам у нас есть закон сохранения импульса, и закон сохранения энергии.
#441 #415274
>>415273
Почему не мяч меняет расстояние между собой и мной?
#442 #415276
>>415274
или например, в меня летит пуля(я бы так сказал исходя из здравого смысла), но сказать, что это я двигаюсь к пуле, а пуля неподвижно будет так же верно.
#443 #415277
>>415276
неподвижна*
#444 #415278
>>415276
Ты не двигаешься никак. Внешние системы никак не влияют ни на тебя, ни на пулю, кроме косвенных факторов.
#445 #415281
>>415278
Минуточку. Движения вообще нет?
#446 #415283
>>415281
Ты ебанат?
#447 #415284
>>415283

>Ты не двигаешься никак


Может ты?
#448 #415285
>>415284
может
#449 #415292
>>415274
Вы оба меняете расстояние между вами.
Кирилл51 Кб, 283x294
Самый тупой вопрос треда #450 #415318
Как смириться с тем, что ты слишком тупой и никогда ничего не достигнешь в науке?
Я только могу смотреть на красивые формулки, восхищаться гением великих людей, а сам нихуя, ппц грустно.
Как же свыкнуться с мыслью, что твоим именем не назовут никакой оператор, формулу, теорему, или хотя бы лемму?!
#451 #415320
>>415319

>Вообще в этом времени уже слажна что либо скаставать, технологии стали слишком ниебически сложными, информации столько что хоть всю жизнь ассимилируй и сотой части всего не усвоишь


Я себя так же успокаиваю.
#452 #415323
>>415318
любой кто достиг что-то пашет, много пашет, пашет с самого детства. Это не ты тупой, ты просто нихуя не делаешь.
Вероятность самозарождения жизни, шах и мат атеисты.1,2 Мб, 1268x2452
#453 #415329
>>415327

>тебя на самом деле нет


:) Хуею с твоей тупезны. Как раз таки твои меня флутуации это чисто гепотетический объект в который я делаю выбор верить или нет, а вот моё существование дано мне непосредство, хотя ты можешь и не верить в то что я есть, это в принципе, возможно. Ой лол, школьник открыл для себя методолАгический авторитет, как это мило, школьник настолько ведом авторитетом что готов отрицать своё существование лижбы бы ему не противоворечить! ВАУ! тебя в РАНе заждались, как раз такие там и нужны. А раз уж ты варишку то открыл, то соизволь доказать что следование маняОкаму отрезает свободу воли.
#454 #415332
>>415326
Он детериминирован, только вот вариант не один, а их много, бесконечно много, и существуют все сразу. Может ты в том варианте, где ты взялся за ум и стал работать много.
#455 #415335
>>415323
Откуда ты знаешь, что я ничего не делаю?

>любой кто достиг что-то пашет, много пашет, пашет с самого детства.


Но не любой, кто много пашет, чего-то достигает.
#456 #415351
>>415340
Люди сидят и поясняют друг другу о квантовых флуктуациях, хотя даже не знают КЭД, вау... настоящий двач. Ну вы и долбоебы блять.
#457 #415354
Что было первым - сознание или материя?
#458 #415356
>>415335
А если не считать просмотр анимы, ты тоже много пашешь? 8 часов каждый день учишься
с интересом учишься, если без то всё в топку, ибо кпд маленький
#459 #415360
>>415354
Смотря что за система.
#460 #415361
>>415326
Вот. Посмотрите-ка. Посмотрите к чему приводит познание мира через двач. Идеальный пример неокрепшего умишки, готового следовать и с пристрастием внимать любому жирному дегенерату в интернетике.
#461 #415367
>>413326 (OP)
Я так понимаю, что в ото материя это не какой то кусок летящий через пространство, а череда событий связанная каким то особым образом друг с другом(какой то разновидностью причинной связи) так что мы можем назвать эту череду событий одним куском материи?
#462 #415369
>>415367
Ты хоть сам понимаешь что несёшь? Что ты пытаешься понять, болезный? Ты школу то хоть закончил? Если да, то мне грустно стало от тебя.
#463 #415370
>>415369
Хочу понять эволюцию понятия материя. В классической физике всё ясно материя это по сути и есть то что под ней понимается обыденно - просто кусок чего то существующий в пустом пространстве. В ото всё становится мутнее, теперь есть пространство время и это одно и тоже.
#464 #415372
>>415370
И в чём проблема, материя из обычной механики, полностью переносится в ОТО. Даже в квантах нет такой проблемы. Проблема возникает в КЭДе и особенно в квантовой теории поля, когда материя=поле
#465 #415373
А разве причинные законы не должны быть отрезаны бритвой Окама? Мы можем воспринять в процессе опыта с А и В, что А расположено над В или находится вправо от В, но не то, что А является причиной В, то есть объективно можно только утверждать: что когда мы полагаем, что А является причиной В то, фактическое наблюдение которое реально имело место заключалось в том, что мы часто наблюдали как В непосредственно или очень быстро следовало за А; но мы не имеем права сказать, что В должно следовать за А или будет следовать за А в будущем. Мы не имеем никакого основания для предположения, что из факта частого следования В за А какая-либо связь допускается. Если не врать и не выдумывать какие то силы которые мы по честному никаким образом никогда не наблюдали, а говорить честно то нужно говорить "Мы часто наблюдали, что явление В следовало за явлением А", всё, мы ничего больше не наблюдали. Идея причинного закона очень психологически и эмоционально удобна но ненаучна. Часто повторяющиеся совпадения А и В не дают никакого основания ожидать, что они будут совпадать в будущем.
#466 #415374
>>415372

>когда материя=поле


Да это тоже интуитивно понятно. Но вот виртуальные частицы - это уже полная мутота.
#467 #415375
>>415372
Проблема в том, что у ньютона метерия существовала как вещь, а у эйнштейна как событие.
#468 #415377
>>415375
Просто точка превращается в мировую линию. Вот и всё. А виртуальные частицы, это не что иное как просто поле частиц, и рано или поздно ты можешь выцепить какую-нибудь оттуда.
#469 #415378
>>415377

>Просто


Ясно :) Хорошо когда в жизни всё просто и думать не надо. Мне бы так.
#470 #415380
>>415378
Думать надо, но явно не над СТО. Над СТО я думал подобно тебе, в классе 8-ом наверное, даже сам вывел пребразование Лореца из предположения что скорость света постоянна. В то время я конечно не знал, что это называется преобразованием Лоренца, был очень рад, когда физик мне объяснил, что я придумал велосипед.
#471 #415381
1 2 3 4 ...
2 4 6 8 ...
Каждому из чисел верхнего ряда соответствует число в нижнем ряду, поэтому число членов в обоих рядах должно быть одинаково, хотя нижний ряд состоит только из половины членов верхнего ряда. Получается не существует бесконечного числа? Бесконечной может быть только совокупность чисел?
#472 #415384
>>415380
Чёт хуйво думал раз недодумал.... и препод не помог, вот оно современное образование. Тугодум учит тугодума думать, тут кроме как на любимого соловецким образованием Гегеля с его отрицанием отрицания дающего синтез на больную величину надеяться не на что, прогресс отменяется. Вспоминаем студентов самого престижного физмата планеты не осиливших построить средневековую катапульту, притом что пользовались они помощью профессиональных рабочих, а не своими руками ничего кроме титьки матери и карандаша с мышкой не державшими.
#473 #415385
>>415384
совецким*
#474 #415386
>>415384
К чему ты вообще? Это ты пытаешься подъебать меня, когда я был в 8-ом классе? Ну молодец вообще, еще что ты можешь? Хочешь расскажу о чём я думал во втором классе, еще посамоутверждаешься.
14830964080400150 Кб, 528x398
#475 #415396
#477 #415425
>>415262

>обмениваться сигналами, то выяснится, что в машине замедлилось время


>время замедлится у тебя


вот этого момента я абсолютно не понимаю. что подразумевается под "замедлением времени"? это типо на уровне мироощущения, или на физическом уровне? более грамотно сформировать свой вопрос не могу, но надеюсь, что ты меня поймешь.
я другой анон, ес че
#478 #415427
>>415425
буквально медленнее.
#479 #415436
>>415427
не понимаю, как это
#480 #415438
>>415436
Если у него будут часы, то они будут медленнее идти.
#481 #415440
>>415438
я не понимаю почему это происходит
#482 #415441
>>415440
Могу на пальцах объяснить. Странным способом (и неверным немного). Другой способ чисто математический, и понять его сложнее будет.
#483 #415442
>>415441
было бы круто и так и так)
#484 #415480
>>414826
Потому что сила, с которой пластина притягивается к магниту, равна силе с которой магнит притягивается к пластине и они разнонаправлены.
>>414837

>чтобы вдавить эту заслонку в условиях притяжения магнитом - нужна ли энергия?


Разумеется нужна. Для перемещения любых тел нужна энергия. Особенно когда эти тела перемещаются в нескольких полях одновременно.
#485 #415482
>>415374
виртуальные частицы - это издержки пертурбативных методов, увы.
#486 #417515
Шар и плита едут навстречу друг-другу. Скорость шара пусть будет В, а скорость плиты- Б. Вопрос. После столкновения шара и плиты массой много больше шара, какую скорость будет иметь последний при абсолютно упругом ударе? Все в неподвижной системе отсчета.
#487 #417570
>>417515
Ну посчитай, это не сложно. (закон сохранения энергии + закон сохранения импульса используй)
#488 #417583
>>417570
только надо не забыть в систему стены перейти, стена с бесконечной массой такое себе
#489 #417585
>>417583
в глаза долблюсь
>>417515
этот анон >>417570 все правильно сказал, потом устремляешь массу стены к бесконечности и получаешь ответ в неподвижной системе
#490 #419860
В чем заключается половой инстинкт у человека? Увидеть своё дитя, или просто выдать/принять сперму? Другими словами, если вырастить людей в дали от цивилизации и стерилизовать их, то будут ли они лезть на стену из-за отсутствия у них детей, или вещи вроде "семья и дети - это щастье" - лишь социальное погромирование?
#491 #420277
>>419860
Ну это в генах прошито и изменяет баланс веществ в организме, впрыскивая порой наркоту из желез.
#492 #420279
>>417515
Шар получит импульс 2m(Б+В), но так как его скорость была В, то скорость после удара станет 2Б+В.
Это задача на теннисную ракетку же.
#493 #420280
>>413326 (OP)
распространяются ли квантовые эффекты(сцепленность, суперпозиция, вот это вот) на радиоволны, ну то есть э/м волны частоты от 100 Гц до 3 ТГц?
#494 #420281
>>414499
вообще-то это как раз следствие ограничения скорости.
>>414493
Сама же скорость света это что-то вроде тактовой частоты Матрицы, пропускной способности эфира, или как-то так. Можно и проще: скорость света в вакууме это просто максимальная скорость распространения э/м волны(в вакууме). А поскольку все состоит в конечном итоге из э/м волн, эта скорость является критической и для материи.
#495 #420290
>>414496
Таки не постулат о постоянстве скорости, конечность-следствие.
#496 #420292
>>420280
Да. Например, одна радиоволна может интерферировать сама с собой в щелях.
#497 #420294
>>420280
Да, только ты их хуй увидишь.
#498 #420296
>>414622
что произойдет со мной, если >оставаясь собой МОЯ ДНК постепенно превратится в кремниевую?
#499 #420297
Что произойдет с фотоном, если его "остановить"?
Я понимаю, что без "взаимодействия" он не "остановится". Но хочу услышать, что произойдет с фотоном, который в рамках мысленного эксперимента потерял скорость без взаимодействия и находится в покое.
#500 #420298
>>420297
Ну, электростатический потенциал по сути это фотон без временной зависимости. Тебе это нужно?
#501 #420299
>>420296

> ДНК


> кремниевую


Это что?
#502 #420300
>>420292
пусть так, но работают ли для радиоволны эти эффекты с "наблюдателем"? ну тоесть если я возле одной из щелей поставлю маленькую антенну, которая не будет глушить волну, будет ли интерференция?
Возможна ли так же квантовая запутанность двух волн, вышедших из одной антенны и как вообще тут представлять фотон?

>>420294
ну почему же, делаешь антенную решетку и смотришь, так же как в видеокамеру.
>
#503 #420301
>>414632
могут, это социальное взаимодействие + контроль рождаемости. У стайных животные все немного сложнее, выживает не только конкретная особь, но и вся стая. По той же причине люди идут в армию, например - добровольно погибая ради своей стаи. Кроме того, геи физически могут размножаться, и при острой нехватке потомства таки станут это делать.
#504 #420302
>>420296
В смысле, структура ДНК в каждой клетке будет заменяться на такой же формы структуру из кремния? Умрёшь. Молекулярные механизмы клетки не смогут снимать РНК-копии с кремниевого памятника ДНК, нет РНК - нет белков, нет белков - клетки разрушаются, отказывают органы, организму конец.
#505 #420304
>>420302

>Умрёшь.


наоборот, кремнийорганика просто не сможет подменить собой РНК, клетка не настолько тупая чтоб всякую хуйню вместо ДНК пихать.
#506 #420329
>>420302
>>420304
А что если сделать некое подобие рибосом, умеющих в транскрипцию органических РНК на матрице из кремниевой ДНК?
А дальше - уже трансляцию белков из них.

Ну или ещё похлеще - репликация ДНК для конструирования ядер стволовых клеток из эталонной кремниевой ДНК,
с последующим митозом ядра и отделением ядер живой стволовой клетки от родительской клетки,
содержащей порождающие ядра с хромосомами и эталонным образцом ДНК из кремния, которое не разрушается.
Таким образом, производя например секвенирование ДНК, и запись их в эталонную кремниевую ДНК,
можно было бы как-бы печатать собственные стволовые клетки неограниченно долго и в промышленных масштабах.
Только аминокислоты подавай и АТФ подавай туда. Хэххэх.
#507 #420330
>>415354
Материя - следствие взаимодействия частиц, сознание - процесс взаимодействия.
мимокун
#508 #420336
>>420297
чтобы его остановить, ты должен сожрать его энергию. А это его убьет.
#509 #420338
>>420290
Да, согласен. конечность следствие. Но почему именно такое значение константы, мы пока не знаем. Просто экспериментальные данные.
#510 #420400
>>420298
>>420336
Время жизни фотона существенно превышает время жизни вселенной, если мне не изменяет память. Но, в рамках мысленного эксперимента, фотон существует без вселенной, предельно упрощенная модель. Есть некое "пространство", есть некое "время", но нет ни взаимодействий, ни полей, ни энергии, кроме энергии одинокого сыча-фотона, теоретически летящего, но в отсутствии точки отсчета - покоящегося, что само по себе оксюморон.

>это его убьет


Прекрасно, "убийство" фотона в коробке - то, что доктор прописал. Что произойдет с фотоном после смерти? На что он "распадется"?
В каком виде будет существовать энергия, выделившаяся в результате его смерти? В виде

>электростатического потенциала?

#511 #420402
>>420299
>>420302
Начало здесь
>>414622

> > Если инопланетный вирус попадёт в клетку, то как он сможет встроиться в ДНК человека,


> >ведь он имеет совершенно иное строение, чтобы потом на его основе с помощью факторов транскрипции создавалась мРНК?


>Он может быть просто повредить клетку, а может заменить её ДНК своей, если например ДНК кремниевая.


>Представь себе, ты попадаешь на другую планету и через время, оставаясь собой твоя ДНК постепенно превращается в кремниевую.


>Почему кремниевая? Да потому что кремний четырехвалентен, как и углерод, и являясь альтернативой ему,


>может образовывать кремнийорганические соединения.

#512 #420410
>>420402
Воу, воу, какая суровая фантастика пошла. Инопланетный кремниевый вирус пронзает клеточную мембрану бластером, пинками разгоняет транскиптазу с интегразой и берёт на абордаж ДНК? Если бы вирусы были настолько суровы, то берёзы бы зимой гриппом болели, а мы бы от табака табачную мозаику подцепляли. Но нет.
Оболочка вируса цепляется к определённым белкам клеточной мембраны. Нет нужных "стыковочных портов" - вирус в пролёте.
Вирусный код использует для встраивания молекулярные механизмы клетки. Удалось "пристыковаться" к клетке и впрыснуть РНК, но обратная транскриптаза не распознала ключевой фрагмент - вирус опять в пролёте.
#513 #420413
>>420402
перейдет в другие виды энергии. Но вопрос в том, а в чём тогда хранятся другие виды энергии? Особенно потeнциальная?
Вспоминая еще о вторичном квантовании, и что вертуальные фотоны есть везде... зачем вообще думать об этом.
#514 #420419
>>420413
Орнул, конечно
pre1485097499wpapersru--[1]127 Кб, 640x400
#515 #420428
>>420410
А что если инопланетный вирус, цепляясь к мембране клетки - производит инъекцию кремниевых нуклеотидов в цитоплазму клетки,
или же образовывает внутри клетки некую неоотторгаемую органеллу,
которая в процессе развития - начинает спамить кремний-органические нуклеотиды,
причём не простые, а комплементарные биологическим,
которые как-бы дополняют нуклеотиды на основе углерод-содержащей ДНК,
из которых ДНК-полимераза конструирует новую кремниевую ДНК в процессе обычной репликации?
Питаться такая хрень могла бы например силикатами или кремниевой кислотой, синтезируемой внутри из микроэлементов.

Тогда, обычная ДНК могла бы быть заменена на кремниевую со временем, без потери функциональности клетки.
Но это просто замена ДНК. А если речь идёт о вирусе, значит может быть и модификация ДНК.
В таком случае, фрагменты из которых ДНК-полимераза конструирует ДНК, могли бы ещё иметь возможность
непосредственно влиять на функции этой полимеразы, как-бы реорганизуя структуру самой ДНК.
А теперь представь что вся эта система ещё и живуча и она изменяется ещё и циклично,
по биоритмам - как у оборотней, лол.
#516 #420433
>>420428
Это тогда не вирус должен быть, а какой-то внутриклеточный паразит типа хламидий или токсоплазмы. Но всё равно непонятно, ради чего кремнийорганический паразит полезет в улеродноорганическую клетку. Это тогда должна быть какая-то планета, на которой параллельно обычная органика сосуществует с кремниевой, и оно всё как-то хитро переплелось со временем.
#517 #420436
>>420433
Ну вообще-то вирусы не являются живыми - это скорее просто структура из ДНК, способная размножаться внутри другой клетки.
А без этих клеток, вирусы просто существуют в виде той же пыли и не проявляют никаких признаков жизни.
#518 #420467
>>420410

>какая суровая фантастика пошла


Вот я и удивился. Задал уточняющий вопрос.
Твоим ответом удовлетворен, анонимус.
#519 #420534
>>420428
Если ты превратишь клетку в кремневую жизнь, то она умрёт. Ведь как известно кремневая жизнь при контакте с водой умирает. А т.к. клетка состоит из воды, то и умрёт она сразу.
Лол. Это я конечно шучу, но умрёт она из за не совместимости с теми химическими процессами которые при нормальных условиях выполняются.
>>420436
Кто-то определил что такое жизнь?
#520 #420573
>>420534

>Кто-то определил что такое жизнь?


Насовское определение гугли.
Тред утонул или удален.
Это копия, сохраненная 23 декабря 2017 года.

Скачать тред: только с превью, с превью и прикрепленными файлами.
Второй вариант может долго скачиваться. Файлы будут только в живых или недавно утонувших тредах. Подробнее

Если вам полезен архив М.Двача, пожертвуйте на оплату сервера.
« /sci/В начало тредаВеб-версияНастройки
/a//b//mu//s//vg/Все доски